Friday, November 30, 2012

Nagel and his critics, Part IV


Continuing our look at the critics of Thomas Nagel’s recent book Mind and Cosmos, we turn to philosopher Alva Noë’s very interesting remarks over at NPR’s 13.7: Cosmos & Culture blog.  Noë’s initial comments might seem broadly sympathetic to Nagel’s position.  He writes:

Science has produced no standard account of the origins of life.

We have a superb understanding of how we get biological variety from simple, living starting points. We can thank Darwin for that. And we know that life in its simplest forms is built up out of inorganic stuff. But we don't have any account of how life springs forth from the supposed primordial soup. This is an explanatory gap we have no idea how to bridge.

Science also lacks even a back-of-the-envelop [sic] concept explaining the emergence of consciousness from the behavior of mere matter. We have an elaborate understanding of the ways in which experience depends on neurobiology. But how consciousness arises out of the action of neurons, or how low-level chemical or atomic processes might explain why we are conscious — we haven't a clue.

We aren't even really sure what questions we should be asking.

These two explanatory gaps are strikingly similar… In both cases we have large-scale phenomena in view (life, consciousness) and an exquisitely detailed understanding of the low-level processes that sustain these phenomena (biochemistry, neuroscience, etc). But we lack any way of making sense of the idea that the higher-level phenomena just come down to, or consist of, what is going on at the lower level.

End quote.  Now an Aristotelian would say that this is precisely what we should expect.  What modern biologists and neuroscientists have uncovered in exquisite detail are the material-cum-efficient causes of the phenomena of life and consciousness.  But that is only half the story, for there are also irreducible final and formal causes -- the inherent teleological features natural objects exhibit by virtue of their substantial forms -- and you are never going to capture those features in terms of material and efficient causality.  That is (one reason) why there always seems to be something left out in materialist accounts of life and consciousness.

There is a mystery here only if you suppose that “lower-level” descriptions are somehow more privileged than “higher-level” descriptions.  And that, we old-fashioned Aristotelians would argue, is something there is no good reason to believe in the first place.  It is merely a metaphysical dogma -- as old as Democritus and Leucippus but no more plausible now than it was in their day -- that is read into the scientific facts rather than read out of them.  In the case at hand, what Noë is describing confirms the traditional Aristotelian view that there is a difference in kind and not merely degree between the organic and the inorganic, and between sensory and vegetative forms of life (in the technical Aristotelian sense of “vegetative,” which does not correspond exactly to the colloquial use of that term).

This has nothing to do with vitalism, “Intelligent Design” theory, and other such bogeymen, and one reason Nagel’s inchoate neo-Aristotelianism may be troubling to his more ideological critics is precisely that it undermines the false dilemma that is the naturalist’s main rhetorical weapon: “Either accept some form of naturalism or you’ll be stuck with magic, obscurantism, or a god-of-the-gaps.”  For though Nagel’s own version is inchoate, neo-Aristotelianism cannot be dismissed as philosophically unserious, and has been worked out in more systematic detail by a number of prominent contemporary philosophers.  (I noted several examples in the first post in this series.  For a recent defense of a neo-Aristotelian position in biology, specifically, see David Oderberg’s Real Essentialism.  I’ve criticized biological reductionism from an Aristotelian point of view in several earlier posts, such as this one, this one, and this one; and neuroscientific reductionism in several other posts, such as this one and this one.)

Now, Noë himself is no ideologue.  This is evidenced not only by the comments already cited, but by his recognition of the depth of the difficulties facing materialism, and of their roots in the very nature of the scientific revolution:

The scientific revolution took its impulse from what the philosopher Bernard Williams called the Absolute Conception of Reality. This is a conception of the world as "it really is" entirely apart from how it appears to us: a colorless, odorless value-free domain of particles and complexes moving in accordance with timeless and immutable mathematical laws. The world so conceived has no place for mind in it. No intention. No purpose. If there is mind — and of course the great scientific revolutionaries such as Descartes and Newton would not deny that there is mind — it exists apart from and unconnected to the material world as this was conceived of by the New Science.

If modern science begins by shaping a conception of the cosmos, its subject matter, in such a way as to exclude mind and life, then it shouldn't come as a surprise that we can't seem to find a place for them in the natural order so conceived.

This is why Nagel observes, at the beginning of his book, that the mind-body problem isn't just a local problem concerning brains, behavior and the mind; correctly understood it invades our understanding of the cosmos itself and its history. 

End quote.  This is a point I’ve emphasized in my own work many times and which (as I’ve emphasized in the earlier posts in this series) has informed Nagel’s own thinking since his article “What Is It Like to Be a Bat?”  Though materialists sometimes suggest that dualism represents a reluctance consistently to follow out the implications of the scientific revolution, the truth is precisely the reverse -- in fact it was the re-conception of matter put forward by the founders of the scientific revolution that led to (Cartesian forms of) dualism.  

Noë even dismisses as “superficial and unsatisfying” the suggestion of critics like Brian Leiter and Michael Weisberg that Nagel’s arguments have little merit given that most philosophers today would probably reject the claim that neuroscience, biology, and chemistry can all be reduced to physics.  For as Noë correctly observes:

[T]here is no stable or deeply understood account of how these autonomous domains fit together. The fact that we are getting along with business as if there were such an account is, well, a political or sociological fact about us that should do little to reassure.

End quote.  As I noted in the two previous posts in this series, the autonomy of these sciences, far from saving naturalism from critiques like Nagel’s, itself only provides further vindication of the holistic Aristotelian account of the natural phenomena studied by these special sciences -- just the sort of position toward which Nagel points, however sketchily.

All the same, Noë resists following Nagel’s call for a radical rethinking of the naturalist consensus.  (And now I get to justify my illustration of Noë as playing Dr. No to Nagel’s James Bond.  All in fun, Prof. Noë!)   Noë proposes instead that:

[T]here is another strategy for responding to the explanatory gaps. This has been one of philosophy's orthodox strategies at least since Kant and it is an approach championed by many of the 20th century's greatest thinkers, from Carnap and the logical positivists down through Wittgenstein and Ryle, to Dennett. According to this strategy, the seeming gaps are, really, a cognitive illusion. We think we can't explain life, but only because we insist on adhering to a conception of life as vaguely spooky, some sort of vital spirit. And likewise, we think we can't explain consciousness, but again this is because we cling to a conception of consciousness as, well, somehow spiritual, and precisely because we insist on thinking of it as something that floats free of its physical substrates ("a ghost in the machine"), as something essentially interior and private. Once we clear away these confusions, so this alternative would have it, we realize that we don't need to solve any special problems about life and mind. There never were any problems.

End quote.  There are several things that can be said in response to this strategy.  For one thing, and as I have already indicated, “vital spirits,” ghosts, and the like are straw men, at least if directed at Aristotelianism.  (Though frankly, they’re not really fair against Cartesianism either, but I’ll let the Cartesians defend themselves.)  It simply is not the case that to reject materialistic naturalism is to opt for magical or otherwise “spooky” forces and entities; it is, rather, simply to opt for an alternative metaphysics (and I have explained the difference between magic and metaphysics elsewhere).  Of course, Noë might not really be suggesting that critics of materialistic naturalism are committed to magic or other pseudo-explanations.  He may merely be suggesting that explanations of a materialistic naturalist sort are preferable to non-materialist explanations, even if the latter are genuine explanations.  But if that is what he means then he is begging the question, since whether materialistic explanations are to be preferred to non-materialist ones is part of what is at issue in the larger debate between Nagel and his critics.  

But put to one side the question of what positive alternatives there might be to the materialistic naturalism that is Nagel’s target -- neo-Aristotelian hylemorphism, Cartesian dualism, vitalism, idealism, panpsychism, neutral monism, or whatever.  Noë’s response would fail even if none of these alternatives was any good.  To see why, suppose that a critic of Gödel's incompleteness theorems suggested that every true arithmetical statement in a formal system capable of expressing arithmetic really is in fact provable within the system, and that the consistency of arithmetic can in fact be proved from within arithmetic itself -- and that Gödel's arguments seem to show otherwise only because of a “cognitive illusion” that makes formal systems seem “vaguely spooky.”

This would not be a serious response to Gödel precisely because it simply does not show that Gödel is wrong but either presupposes or merely asserts that he is wrong.  Gödel purports to demonstrate his claims.  Hence, adequately to answer him would require showing that there is something wrong with his attempted demonstration, not merely staking out a position that assumes that there is something wrong with it.  Similarly, many of the key arguments against materialistic naturalism -- Chalmers’ “zombie argument,” Jackson’s “knowledge argument,” Ross’s argument for the immateriality of thought, etc. -- purport to demonstrate that materialistic naturalism is false.  Adequately to answer them requires showing that there is some error in the attempted demonstrations, and the appeal to an alleged “cognitive illusion” simply assumes this without showing it.  It merely begs the question.

Furthermore, there would only be pressure to take the “cognitive illusion” suggestion seriously if we had independent reason to think that materialistic naturalism simply has to be right.  And there is no such reason.  Its defenders often point to the “success” of materialistic explanations as reason to think materialistic naturalism is true, but as I have pointed out many times (e.g. here), this sort of argument, however popular, is blatantly fallacious.  To argue:

1. The predictive power and technological applications of materialistic modes of explanation are unparalleled by those of any other purported source of knowledge.

2. Therefore what materialistic explanations reveal to us is all that is real.

is as silly as arguing:

1. Metal detectors have had far greater success in finding coins and other metallic objects in more places than any other method has.

2. Therefore what metal detectors reveal to us (coins and other metallic objects) is all that is real.

Metal detectors are keyed to those aspects of the natural world susceptible of detection via electromagnetic means (or whatever).  But however well they perform this task -- indeed, even if they succeeded on every single occasion they were deployed -- it simply wouldn’t follow for a moment that there are no aspects of the natural world other than the ones they are sensitive to.  Similarly, what materialistic explanations do is to capture those aspects of the natural world susceptible of a materialist analysis -- breaking down larger systems into component material parts, mathematically modeling the parts and their combinations, testing the predictions that follow from these models, and so forth.  But here too, it simply doesn’t follow for a moment that there are no other aspects of the natural world. 

Suppose someone beholden to the idea that coins and other metallic objects are all that exist was confronted with all the obvious counterevidence -- trees, rocks, people, animals, glass, plastic, and all the other non-metal objects there are.  And suppose he acknowledged that there is an “explanatory gap” here but that it rested on a “cognitive illusion” that made trees, rocks, etc. seem “vaguely spooky” insofar as they appeared to “float free of their metallic substrates.”  Of course, no one would take such an absurd suggestion seriously for a moment.  But neither is there any non-question-begging reason to take seriously the suggestion that all the counterevidence to materialistic naturalism rests on a “cognitive illusion.”

As E. A. Burtt warned in his classic book The Metaphysical Foundations of Modern Physical Science, those impressed by the methods of modern science are prone to “make a metaphysics out of [that] method,” to judge reality by the method rather than judging the method against reality.  That is in fact the “cognitive illusion” operative in the debate between materialistic naturalism and its critics, and it seems it is an illusion to which even a reasonable man like Noë might be subject.

219 comments:

  1. http://www.canada.com/technology/Canadian+scientists+create+functioning+virtual+brain/7628440/story.html

    That may pose a problem for dualist theories of the mind. From the article:

    "Spaun's cognition and behaviour is very basic, but it can learn patterns it has never seen before and use that knowledge to figure out the best answer to a question. "So it does learn," says Eliasmith."

    That seems to me that the computer program is displaying proto-intentionality and that it can do some proto-reasoning.

    ReplyDelete
  2. very good post, Dr. feser.

    ReplyDelete
  3. Good post, but why must you upload a picture of scantily clad women striking seductive poses?!

    And that too above a post concerned with classical virtues!

    ReplyDelete
  4. Guess it never occurred to me that some tiny monochrome drawings from a 1962 movie poster might drive some one wild.

    But then, some of the table legs in the house here are uncovered too, so what do I know?

    ReplyDelete
  5. Anonymous, anonymous... I was a programmer for 20 years. This is all rubbish: "Spaun's cognition and behaviour is very basic, but it can learn patterns it has never seen before and use that knowledge to figure out the best answer to a question."

    By "learn," they mean, "the circuitry of the program is reconfigured." By "figure out," they mean "get the right output based on some input. Whether those things constitute "learning" and "figuring out" is the very question on the table!

    ReplyDelete
  6. Gene,

    Pardon the side-question, but you're a panpsychist, right?

    ReplyDelete
  7. Ed,

    1. The predictive power and technological applications of materialistic modes of explanation are unparalleled by those of any other purported source of knowledge.

    I think part of the problem is that people (including yourself) call these things 'materialistic modes of explanations' - but are they really? If an Aristotilean describes the operation of a machine he made, is he being a materialist then and there? It seems to me he's just using a form of explanation that's every bit as compatible with Aristotileanism as it is materialism and even idealism. Describing it as 'materialistic modes of explanation' makes it sound as if it's a kind of explanation that's unavailable to the Aristotilean, as if being able to describe a machine as a machine was itself something an Aristotilean didn't expect was possible.

    ReplyDelete
  8. I have trouble with Feser’s metal detector analogy. I could be remembering incorrectly, but doesn’t Feser say in TLS that empirical investigation has the capacity to replace final causes with material causes? The metal detector merely registers "neutral" when it's not detecting metal.

    (I'm moving house, so I don't have my copy of TLS at hand)

    Surely at this point the naturalist might say that this is a “concession” and that if empirical investigation can ever show that final causes aren’t present at some level of nature, then the Inexorable March of Science or will take care of any other proposed examples.
    How might an Aristotelian respond to this?

    ReplyDelete
  9. Gene, did you read the article? Spaun is not a computer program made from scratch. It's a computer simulation of neurons that are arranged in a similar manner to the way they are arranged in the human brain. Still, the question is whether or not Spaun is displaying intrinsic intentionality.

    ReplyDelete
  10. Professor Feser Lecture at Gonzaga University: http://m.youtube.com/watch?v=_1Dkp1U9pek Part 2: http://m.youtube.com/watch?v=-O40N4nNGUc. I figured that these lectures flew under the cyber-radar somehow , due to the scandalously feeble amount of views registered (Professor Feser's name ought to have been injected into the title somewhere).

    Hey Professor Feser. I'm the kid from the Steubenville Conference that sells Philosopher's autographs on Ebay (inside "joke" from said Conference).

    ReplyDelete
  11. Cale,

    I don't think that's correct. In fact I specifically recall that Ed argued that empirical investigation could not overturn these issues, which had to be handled on a metaphysical/philosophical level.

    ReplyDelete
  12. Non-mobile links to those lectures posted by Anon:

    http://www.youtube.com/watch?v=_1Dkp1U9pek

    http://www.youtube.com/watch?v=-O40N4nNGUc


    ReplyDelete
  13. The Profeser: But then, some of the table legs in the house here are uncovered too, so what do I know?

    I'm shocked. Next thing we know, you'll be listening to jazz.
    But Anonymous apparently didn't like the picture for the post on virtues either, which was an excellent depiction, I thought.

    ReplyDelete
  14. Anon 1

    No necessarily... Dualism in Descartes Terms with at least two substances to create the desired effects may have a problem, but certain dualists approachs like the one of Aristoteleans don't necessarily posit two substances so a computer could achieve a certain amount of similarity to natural brain's functions.

    Now about he proto-abilities of the machine, yeahhhhhh ... well I bet this is the scientists personal metaphysics being read on the experiment( if we defend science as naturalistic where no intentionality exists to begin with ), ORRRRR the human brain can actually detect intention, although I have no idea if computers have any intention per se.

    ReplyDelete
  15. Mr. Green, don't you see? Can't you connect the dots? Feser, in putting forth pictures that treat women as objects rather than as subjects, perpetuates an objectification of women in the social consciousness, and is thereby doing his share in promoting promiscuity and RAPE CULTURE!

    ReplyDelete
  16. I thought it was supposed to be based on 007's first movie Dr No...

    You people need culture U_U!!! CULTURE I SAY!!!!

    ReplyDelete
  17. "Suppose someone beholden to the idea that coins and other metallic objects are all that exist was confronted with all the obvious counterevidence -- trees, rocks, people, animals, glass, plastic, and all the other non-metal objects there are. And suppose he acknowledged that there is an “explanatory gap” here but that it rested on a “cognitive illusion” that made trees, rocks, etc. seem “vaguely spooky” insofar as they appeared to “float free of their metallic substrates.” Of course, no one would take such an absurd suggestion seriously for a moment. "

    Don't materialists take as their starting point that tress, etc., don't really exist and are really just metal after all? In their dismissal of 'tree' as mere convenient label and reduction of all matter to quarks or whatever is the current smallest sub-atomic unit, isn't it precisely their intention to deny the existence of trees? I think many people do take that absurd suggestion seriously.

    What I have never understood is why, if 'essence' is an invalid concept, mind and trees are out, but quarks and electricity get a free pass. The fact of the communicability of ideas through language seems to refute pure materialism and prove the existence of essence as easily as Johnson's kicking the stone refutes idealism.

    ReplyDelete
  18. Why does kicking the stone refutes idealism???

    Why communication proofs essences???

    *Those are interesting and being as philosophically illiterate as I am, I would love to hear something like that to be less like Linton and more like ... My idealized version of me.*

    Oh but is not materialism that is: physicalism! THE NEW SPIN-OFF of the same old same old.

    ReplyDelete
  19. Eduardo,

    Actually, Cartesian Dualism wouldn't really have a problem with it either. There's no real reason to posit that machines can't ever have information about the world so that they can navigate it. Most of the latest dualist comments, starting from Nagel and through Chalmers and Jackson, are all about phenomenal experience, or an actual experiential point of view. If there's anything that we've discovered, it's that you don't always need a phenomenal point of view to be able to act appropriately towards things. Consciousness, then, is critically attached to having this point of view, and being able to react appropriately to the environment doesn't require this point of view necessarily. For me, I see phenomenal experience as one sort of input, and that input can be replaced with other sorts of input ... and then in us that input can be something non-physical (whatever that means).

    About the specific case, though, this sounds an awful lot like connectionist systems, and the question is not really over whether they learn -- because there are certainly definitions of "learn" under which they do learn -- but whether they UNDERSTAND. There is a difference between simply reciting an answer off by rote and answering because you understand what's going on. Connectionists systems tend to fail miserably at understanding; they can learn to give the right answers, but if you step outside that rote learning they tend to fall apart very quickly.

    ReplyDelete
  20. you mean like the Chinese room or something???

    Well true I didn't really thought about that in the very start but I had something else in mind...

    Anyways my intention was to reply to the usual sweeping assertion, you know the thing that all comboxes are filled with ... assert with no argument whatsoever then just pretend you did some AWESOME argument XD.

    I just still believe I can fix that in people U_U, yeah damn deluded stupid high school education coupled with my weak personality.

    ReplyDelete
  21. Dr Feser

    What? No comment/ no acknowledgement on your plagiarizing Martin Barrack's work in the previous OP [HERE] on Cardinal Virtues, Prof Feser?

    One could argue this was a small transgression of little consequence. But the sentence, "They are so called because they are traditionally regarded as the “hinge” (cardo) on which the rest of morality turns", was no ordinary tittle of information, appropriated without attribution, buried deep within the body of your treatise. This sentence was a framing statement. It was the core around which the framework of the OP on cardinal virtues was constructed. It defined the substance of your argument.

    It would seem fair and reasonable that the appropriate citation and attribution, however belated it be, would be the honourable thing to do. Nothing as yet.

    ReplyDelete
  22. Moving forward. On the matter of Nöe and Nagel,

    "It simply is not the case that to reject materialistic naturalism is to opt for magical or otherwise “spooky” forces and entities; it is, rather, simply to opt for an alternative metaphysics."

    That is pretty much what it is to reject materialistic naturalism [MN]. MN may well be a position characterised as a 'work in progress', but there is no current metaphysical alternative that meshes best with the evidence from the sciences. It is timely reminder that at the most fundamental and practical level, there are two forms of philosophy, scientifically informed philosophy [SIP] and scientifically uninformed philosophy [SUP].

    SUP, continues the tradition and its long-standing relationship with theology. Its underlying operant milieu remains largely predicated on teleology, " .. [a] philosophical account that holds that final causes exist in nature, meaning that design and purpose analogous to that found in human actions are inherent also in the rest of nature." [All references Library] Theology carries that ideation further, by positing that a god is the 'cause' of that teleology. SUP is a highly speculative and suppositional form of philosophy at best, its contemporary intellectuals including: Freddoso, Geisler,Plantinga. Feser, WLC, Oderberg, Moreland, among others.

    In contradistinction, SIP, represents a philosophical trend that grew out of the enlightenment, at a time that it cleaved as a separate discipline to that of theology and grounding its operant system of study in the quantum of scientific evidence and knowledge. "The Enlightenment begins with the scientific revolution of the sixteenth and seventeenth centuries. The rise of the new science progressively undermines not only the ancient geocentric conception of the cosmos, but, with it, the entire set of presuppositions that had served to constrain and guide philosophical inquiry. The dramatic success of the new science in explaining the natural world, in accounting for a wide variety of phenomena by appeal to a relatively small number of elegant mathematical formulae, promotes philosophy (in the broad sense of the time, which includes natural science) from a handmaiden of theology, constrained by its purposes and methods, to an independent force with the power and authority to challenge the old and construct the new, in the realms both of theory and practice, on the basis of its own principles." Stanford Encyclopedia of Philosophy [http://plato.stanford.edu/entries/enlightenment/] Scholars representing this philosophical stream, Drange, Dennett, McGinn, Oppy, Popper etc.

    Nagel and Nöe reside somewhere on the fence largely unable to slough off the proclivity for imagining agency where there is none, and unable to intellectually check the pull of our evolved predilection towards unschooled and spontaneous notions of teleological intentionality.

    This OP, emblematic of SUP, is largely an inconsequential exercise because, in the end, (as one commenter noted on Nöe elsewhere) 'most of what he [Nöe] talks about is understood and rather well explained by science and does NOT require anything more than the brain.' And as another noted, 'he's not actually a neuroscientist... he's a neurophilosopher.'

    ReplyDelete
  23. Let me see here...

    Wishful thinking, followed by irrelevance to the subject, and sweeping assertions without any arguments.

    Wow paps you are my hero ... now what about drink the Kool-aid somewhere else huh?

    ReplyDelete
  24. Thanks paps you along with me killed the combox... I hope you are happy whatever happiness might be in your world!

    ReplyDelete
  25. "Spaun's cognition and behaviour is very basic, but it can learn..."

    This is called: "anthropomorphic fallacy".

    ReplyDelete
  26. "If there's anything that we've discovered, it's that you don't always need a phenomenal point of view to be able to act appropriately towards things."

    And people say that there are no final causes in nature. Also, assuming Aristotle's four causes are true, is there anything stopping us (metaphysically) from creating a machine that possesses intrinsic intentionality? Also, from a summary of some recent naturalism conference, it seems like some of them have adopted externalism to explain intentionality.

    ReplyDelete
  27. Externalism ???

    something like the intention only exist on the observer and not on the object or something like that?

    Well can machines really have intentions? Or do they simply go from one state to the next and we interpret the relation of input and outputs as intention?

    Maybe a computer that is not so much like a computer can have intentions...

    ReplyDelete
  28. Papalinton,

    One could argue this was a small transgression of little consequence. But the sentence, "They are so called because they are traditionally regarded as the “hinge” (cardo) on which the rest of morality turns", was no ordinary tittle of information, appropriated without attribution, buried deep within the body of your treatise. This sentence was a framing statement. It was the core around which the framework of the OP on cardinal virtues was constructed. It defined the substance of your argument.

    It would seem fair and reasonable that the appropriate citation and attribution, however belated it be, would be the honourable thing to do. Nothing as yet.


    You are really embarrassing yourself, friend. In the interest of both reputations at stake here, we can see that this idea/formulation/prose is too common for Mr. Barrack to own. From just a cursory look:

    Wikipedia:
    "The term 'cardinal' comes from the Latin cardo or hinge; the cardinal virtues are so called because they are hinges upon which the door of the moral life swings."

    Right and Reason, Fagothey:
    "Four virtues have been traditionally picked out as the most important in the ethical order. They are called cardinal virtues, from the Latin cardo, a hinge, because they are the four hinges on which the other virtues swing."

    An Introduction to Philosophy, Sullivan:
    "The moral virtues, prudence, justice, temperance, and fortitude, are called cardinal (from the Latin word meaning 'hinge') because they are virtues 'on which the moral life turns and is founded.'"

    et cetera, et cetera...so just take it easy there with the plagiarism talk.

    ReplyDelete
  29. What? No comment/ no acknowledgement on your plagiarizing Martin Barrack's work in the previous OP [HERE] on Cardinal Virtues, Prof Feser?

    Really?

    You're being serious? This isn't a joke?

    Papalinton, to call you unhinged would be false only insofar as it might insinuate that you once had been "hinged."

    From now on, folks, please regard Papalinton the way you would regard TruthOverfaith or the one or two other psychotics who've been banned from the blog. Since exchanges with such people have no intellectual value whatsoever but simply spam up the combox and discourage worthwhile commenters, they will in future be deleted.

    Papalinton, you are now officially invited to get lost.

    ReplyDelete
  30. Papalinton could make a pretty penny in the entertainment industry.

    ReplyDelete
  31. Great article. But I think it's clear that Noë is an intelligent man. Unlike most naturalists, he actually understands his position and its problems. I'm just not sure that he understands the eliminativism that he implicitly endorses, or how pathetic Dennett's arguments for that eliminativism really are.

    ReplyDelete
  32. Re Dr Feser and plagiarism. Not correct.
    Egg all over my face.

    ReplyDelete
  33. Let me Echo those words Paps just in case you haven't heard them...

    "Papalinton, you are now officially invited to get lost."

    But look for the Bright side, you can go back to your homies! They agreee with everything you say, you won't have to waste time with the illogical stupid arseholes around here no more, soooo...

    "Papalinton, you are now officially invited to get lost."

    ReplyDelete
  34. @Edward Feser:

    Thanks for clearing up the atmosphere. The comment box was getting to be a seriously unwholesome place with that Troll around.

    Now, how about that other loony? (crosses fingers, pleading eyes)

    ReplyDelete
  35. >> But then, some of the table legs in the house
    >> here are uncovered too, so what do I know?

    > I'm shocked.

    Why? The sole cause for concern vis-a-vis uncovered table legs is that, due to poor ciculation stemming from standing still for long periods of time, they may develop varicose grains--the unsightliness of which happens to be easy to address.

    ReplyDelete
  36. Well grodrigues ...

    I am going nowhere!!!! ò_ó

    ReplyDelete
  37. Papalinton wrote:

    Re Dr Feser and plagiarism. Not correct.
    Egg all over my face.


    "Oops, egg on my face" is something usually said by someone who makes an honest mistake.

    Not by someone who deliberately, aggressively, and repeatedly rubs egg onto his face and loudly asks everyone to come look at it.

    You're turning the combox into a freak show and I've already tolerated your ridiculous antics for too long. Get lost. You're banned.

    ReplyDelete
  38. Hello Dr. Feser,

    I was wondering if you could direct your readers attention to this http://www.blackstonefilms.org/films/unnatural-law/unnatural-law.html It's a documentary that plans to explain the Church's teaching on homosexuality sans the bias of the secular media. They are about $17,000 short of their fundraising goal to start production and they only have 7 days left to raise the remainder of the money. Sorry about this being off topic, but I thought the only way to bring it to the attention of a substantial amount of people was to say something about it here.

    ReplyDelete
  39. grodrigues wrote:

    Now, how about that other loony? (crosses fingers, pleading eyes)

    We'll see. In general I prefer not to ban anyone merely for the content of his views, however insane or offensive. When I have banned people, it's been for their demonstrated inability to meet even the basic conditions for rational combox discussion and a tendency to drag the whole combox down as a result -- people who only ever post insane or obscene comments, people who are relentlessly rude, people who spam up the combox with frequent logorrhetic comments (especially of a nature irrelevant to the post commented on), etc.

    The person I assume you have in mind comes closest to the third sort of case -- too frequent logorrhetic comments -- but he has stopped doing this when I have asked him to. And as far as I know -- I read very little of what he posts -- he is not rude, insane, or obscene in a Perezoso or TruthOverfaith sort of way. So to this point I have not been inclined to ban him.

    At the end of the day, though, it really is up to other commenters to self-police. If people (including me on one occasion) had just ignored Papalinton's repeated idiocies, I probably would not have to have banned him. If you or anyone else thinks some other non-banned commenter is not worth engaging, then simply stop engaging him.

    ReplyDelete
  40. Dr Feser, Paps idiocy is what makes me wake up every day and have strenght to live my life... I woke up and go: "Damn, there is someone way more stupid than me out there and he doesn't even CARE!".

    Then I go to do whatever my crazy head has in it!!!

    So paps is g.... oh damn it, can't say that even in sarcastic mode.

    ReplyDelete
  41. Ah, here it is. Finally found it, yes I did. It is part of a post I made to a blog back on June 10, 2010:

    - - - - -
    Browsing in a bookstore earlier today, one book stood out for its title. The title of the book is _Out of Our Heads_, and is written by a philosopher. Contrary to what the title might seem to imply, however, the book is not a whistle-blowing expose written by an insider from the community of philosophers. Rather, it pits two views regarding consciousness one against the other, then advances a third view.

    The first view of consciousness, the older view, is that it is something immaterial or spiritual flowing into and affecting us. The second view of consciousness, the newer view, is that it is merely a by-product of the physiological, biochemical processes of a brain in action. (This latter view is sometimes summed up as 'the mind is what the brain does'.) The author notes these two views of consciousness, then advances a third view.

    The third view, basically, is that consciousness is neither the product of something flowing in and affecting the brain, nor the buzzing hum of the feverishly working machinery of the brain, but what we ourselves (who, though we have a brain, are not our brains) do.

    It can be said that the first and second views of consciousness are at odds with one another mainly because different meanings are assigned to the same term. And it can be said that the author apparently feels that the best way to resolve the dispute is to come up with and assign a third different meaning to the overworked term.

    Writes the author, I use the term "consciousness" to mean, roughly, experience. And I think of experience, broadly, as encompassing thinking, feeling, and the fact that a world "shows up" for us in perception.
    - - - - -

    The author, of course, is Alva Noë. And the full title of the mentioned book is Out of Our Heads:
    Why You Are Not Your Brain, and Other Lessons from the Biology of Consciousness
    .

    Dr. Feser quotes Noë in part as follows:

    ...we think we can't explain consciousness, but again this is because we cling to a conception of consciousness as, well, somehow spiritual, and precisely because we insist on thinking of it as something that floats free of its physical substrates ("a ghost in the machine"), as something essentially interior and private. Once we clear away these confusions, so this alternative would have it, we realize that we don't need to solve any special problems about life and mind. There never were any problems.

    And Noë completes his "very interesting remarks over at NPR’s 13.7: Cosmos & Culture blog" in this manner:

    It is one thing to say that we don't need to solve the problems, that we need to dissolve them. And it is another to deliver the intellectual goods. Nagel's worthwhile book reminds us that haven't done this yet. And that we must.

    And because we must, Noë (apparently) has been seeking to muster a consensus that there are no special problems about life and mind (and never have been)--once we acknowledge consciousness as being, roughly speaking, nothing more than the aggregate of the facts that: a) we think; b) we feel; c) we have perception; and d) the world somehow just "shows up" for us in this perception of ours.

    Groovy, man!

    As Gene noted earlier, By "learn," they mean, "the circuitry of the program is reconfigured." By "figure out," they mean "get the right output based on some input.

    ReplyDelete
  42. And as Germaine Greer once wrote, It is the fate of euphemisms to lose their function rapidly by association with the actuality of what they desginate, so that they must be regularly replace with euphemisms for themselves.

    ReplyDelete
  43. Christian,

    Good luck with the film. Are you putting that on kickstarter?

    ReplyDelete
  44. Crude,

    It's not my film, it's being produced by Fr. John Hollowell. If you follow the link it says in the text that they plan to put it on the web for free and in DVD form, so I'm not sure if it'll be on kickstarter or not. I hope they pass the fundraising threshold, I urge you to donate if you can, I hope more people will take notice of it.

    ReplyDelete
  45. This is slightly beside the point, but I couldn't think of anywhere to put it other than the most recent post.

    You've written about the recent revival of Aristotelian in academic philosophy but would you put Object Oriented Ontology in that camp? Graham Harman basically calls it "weird Aristotle."

    What do you think about that whole movement in general?

    ReplyDelete
  46. Here is a thought?

    THE LAST SUPERSTITION:The Movie.

    ReplyDelete
  47. Casey,

    Looking up object-oriented ontology, it sounds like a lazy blend of Heidegger and panpsychism. Given that both systems are incompatible with (and the former outright hostile to) Aristotelianism, I'm not sure what Harman is talking about. Could be wrong, though.

    ReplyDelete
  48. The world consists of processes and structures.

    The physics-based sciences construct their models, understanding and explanations by abstracting and reducing the numerous natural instances of processes operating on structures, into a few generic procedures operating on data.

    Hence physical explanations will be impossible to construct, will fail, or will be innapplicable as 'category errors' for any phenomena where

    (i) Processes cannot be reduced to procedures
    (ii) Structures cannot be reduced to data

    I suspect that one aspect of The Hard Problem is that some of the processes of consciousness are not even in principle reducible to procedures (they are 'non-algorithmic'). Consequently, attempts at physical explanantions may be a category error. The non-physical mind will forever remain a riddle, wrapped in a mystery, inside an enigma

    ReplyDelete
  49. Can structures ever be reduced to data???

    Unless you mean all sorts of data... but wait ... can it?

    ReplyDelete
  50. Structures can be regarded as 'isomorphic' with data, e.g. the structure of DNA is isomorphic with the AGCT nucleotide alphabet. The sructure of the solar system is isomorphic with a table of planetary masses and orbit dimensions etc.

    ReplyDelete
  51. Oh so you mean that is NOT EQUAL but rather WELL DESCRIBED by the data?

    Like there is a good correlation between pretending at a certain point that worlds are dots moving under a group of Rules but worlds are not really dots.





    ReplyDelete
  52. @ Eduardo
    'WELL PREDICTED' might be the best way of putting it. It is dubious whether data and procedures are capable of describing or being 'about' anything, as intentionality is lacking.

    " All meaning is stripped out of an algorithm before it can be processed by a machine, an operation known as compilation. For example, the following two statements reduce to exactly the same algorithm within the memory of a computer

    (i) IF RoomLength * RoomWidth > CarpetArea THEN NeedMoreCarpet = TRUE

    (ii) IF Audience * TicketPrice > HireOfVenue THEN AvoidedBankruptcy = TRUE

    both, when compiled, will appear to the machine as an anonymous executable form such as
    IF a*b > c THEN d=1

    The algorithmic models for carpet laying and theatrical success are said to be 'isomorphic' - they both have the same form, though they are 'about' very different phenomena. This 'aboutness' is called 'intentionality' by philosophers, and is something that minds can process and machines can't.

    The computer will perform the same internal operations whether its consequences are a visit to the carpet store, or an embarrassing surplus in Max Bialystock’s bank account.
    - http://rational-buddhism.blogspot.com/2012/02/church-turing-deutsch-principle-and.html

    ReplyDelete
  53. Hmmm I guess I won't really argue any longer, we are back at an old subject that has been discussed a lot around here n_n.

    But I really wonder what else was stripped out of the experience or of reality before we are able to turn it into a working algorithm.

    ReplyDelete
  54. But I really wonder what else was stripped out of the experience or of reality before we are able to turn it into a working algorithm.

    This is a good question, and may be rephrased as, "How many necessities have been stripped out of the represented endeavors in order to turn the formals into viable algorithmic models for success in those endeavors?"

    Relying on the first formula, making sure you have enough carpet constitutes a viable algorithm for laying it. And relying on the second formula, one may conclude that, as long as revenue is greater than the particular expense of revenue cost, theatrical success is realized when total expenses exceed revenue.

    It nonetheless remains true that the two example algorithmic models are, in some sense, isomorphic--they both fail.

    But perhaps the real point of the example 'algorithmic models' was that a single formula can have multiple uses.

    ReplyDelete
  55. "This is a good question, and may be rephrased as, "How many necessities have been stripped out of the represented endeavors in order to turn the formals into viable algorithmic models for success in those endeavors?"

    That is enough Glenn it will take me ages to decipher this one XD. But yeah I could rephrase that way is just that is way too damn good of a phrase to waste hahahahhaa.

    Well so what should we conclude about algorithms, they obviously have no will of their own to get it right most likely because they have idea what right or wrong it. In a anthropomorfic sort of way I mean.



    ReplyDelete
  56. A single formula can have multiple uses because no computational model of the mind (or attempted implementation of such a model) contains within itself any intentionality. All such meaning must be projected on to the data and procedures from 'outside the system'. You can have syntax but not semantics within such a model (Chinese Room).

    Now if we assume that computationalism is equivalent to physicalism (Church-Turing-Deutsch principle), we can similarly critique any mechanistic model or explanation of the mind.

    ReplyDelete
  57. what should we conclude about algorithms, they obviously have no will of their own to get it right most likely because they have idea what right or wrong it. In a anthropomorfic sort of way I mean.

    o Algorithms can be useful when properly fashioned.

    o Algorithms can be dangerous when poorly fashioned.

    o Creating an algorithm can be quite challenging, even for tasks which seem to be simple.

    o It is often easier to use an algorithm already existing for a particular purpose than it is to create a new algorithm for that purpose.

    o Etc. etc., etc.

    ReplyDelete
  58. A single formula can have multiple uses because no computational model of the mind (or attempted implementation of such a model) contains within itself any intentionality.

    I think there may be other reasons why a single formula can have multiple uses.

    But I'm guessing that what you mean to say that it is because a single formula can have multiple uses that no computational model of the mind...

    ReplyDelete
  59. How does Aristotelianism explain how, for example, neurons produce consciousness? Is it just 'metaphysically explained' with the actual working details still left to science, and if so, doesn't this beg the question against naturalism, since naturalism metaphysically depends on the results of science?

    Aristotelian metaphysical or 'intentional' explanations don't seem to lead to any 'working' knowledge - they also don't tell us 'how' life arose from inorganic matter, how consciousness arose from matter, why polar bears are white, etc. If Aristotelianism provides a fuller understanding, why doesn't that fuller understanding then lead to discovery of the working mechanisms, etc? It doesn't seem to me that it solves the 'hard problem' at all, except perhaps in a non-overlapping magisteria kind of way.

    Apologies for the inchoate nature of the questions - this is not an attack on A-T or philosophy or metaphysics, just an attempt to understand.

    ReplyDelete
  60. if we assume that computationalism is equivalent to physicalism (Church-Turing-Deutsch principle), we can similarly critique any mechanistic model or explanation of the mind.

    I think more would be needed--if I understand it correctly, the simplified, basic claim wouldn't be that intentionality comes from algorithms which themselves are devoid of intentionality, but that it emerges from the interaction of said algorithms, i.e., it would be loosely akin to a chemical reaction between two inert substances.

    ReplyDelete
  61. "How does Aristotelianism explain..."

    Actually you are just saying that you are waiting for a mechanistical explanation so you are begging the question unwantedly, in your question. What has been metaphysically explained if premises are correct will be explained by metaphysics because science will be worried to explain other stuff. See your question sort of takes in consideration that Science is the only one to truly answer anything because it offers mechanisms as explanations.

    "Aristotelian metaphysical or 'intentional' explanations don't seem to lead to any 'working' knowledge..."

    If that you mean that Aristotelianism doesn't offer an algorithm or a set of rules for you to work with... well in a sense it does, because between the choices of having no intentionality and haveing intentionality, intentionality explains certain things better, or would rather take into account certain things better. About the algorithm ... dunno, can those traits be treated in terms of algorithms?

    Wellll first, there is no problem in critizing XD, even if it is in questions. If you do care about the subject then I guess there is bunch of Thomistic wackos around here to help you! Links, boosk that you might wanna read so on and so on.

    ReplyDelete
  62. "How does Aristotelianism explain how, for example, neurons produce consciousness? Is it just 'metaphysically explained' with the actual working details still left to science, and if so, doesn't this beg the question against naturalism, since naturalism metaphysically depends on the results of science?"

    It's actually hard to provide a succinct reply to this. You'd have to go over act and potency, the four causes, the problems facing materialism, arguments for dualism (properly understood), the real differences between metaphysics, philosophy of nature and science and so forth. If you have (a lot of) time, check out:

    http://edwardfeser.blogspot.com/2011/05/mind-body-problem-roundup.html

    ReplyDelete
  63. "since naturalism metaphysically depends on the results of science?"

    Well actually this is not really true, Naturalism is still metaphysics. let's put this in a simple way.

    Imagine 1 scientist doing a treatise about neurons using A-T and 1 scientist using Naturalism. They just pressupose their metaphysical theories/propositions are correct, and will analyse the neurons, their relations to the surrounding environment, it's chracteristics all by their metaphysics...

    So in the end of day the naturalist doesn't reallly need science to explain anything, everything has already been given a number of elements to explain them and all one looks for in science is the confirmation of a certain model(doesn't have to be necessarily this way), that is completely immersed in his own metaphysics.

    Unlesss... his metaphysics somehow depends entirely on experience or something ... Then their work will resemble more like someone designing a problem-solver of some sorts, where principles get thrown away for all sorts of reasons during production.

    ReplyDelete
  64. Thanks for the link, Black Luster. I have read a number of those posts, although I need to read them again (and again) so I've bookmarked it. I've been reading this site for a while now and I am very compelled by the metaphysics presented, even though I am at the moment an atheist/naturalist/materialist or what have you. But this is a gap for me (not saying it's a correct or good gap), the fact that A-T doesn't seem to lead to an increase in •scientific• understanding - it doesn't appear to provide an explanation of how neurons produce consciousness or experience, etc., yet it egregiously faults naturalism for this. I know it's faulting naturalism for this •metaphysically• and not merely empirically, but I guess I'd like to know why a putatively superior metaphysics doesn't seem to solve the empirical part of the problem of consciousness (or lead to its solution, rather). I think that is one reason, right or wrong, for the 'science works' argument. But perhaps I'm just mixing domains that shouldn't be mixed.

    ReplyDelete
  65. @Glenn
    "I think more would be needed--if I understand it correctly, the simplified, basic claim wouldn't be that intentionality comes from algorithms which themselves are devoid of intentionality, but that it emerges from the interaction of said algorithms, i.e., it would be loosely akin to a chemical reaction between two inert substances."

    Emergent phenomena arise in the mind of the observer. Emergence presupposes the pre-existence of mind and intentionality... http://seanrobsville.blogspot.com/2009/10/consciousness-and-mind-as-emergent.html

    ReplyDelete
  66. Anon

    you seem committed with mechanistical view of the world and of science in general... the problem lies in you!

    Not trying to judge you or anything, just saying it a case of one's cherished premises that guides one's epistemology

    ReplyDelete
  67. Eduardo, your replies, except for your last, are mostly unclear to me. No doubt the problem there is mine also. Sorry.

    ReplyDelete
  68. I ... have failed once more ... u_u all I am good for is to make fun of the usual trolls, so ashamed.

    But if at first you don't succeed.

    You seem to correlate mechanism with explanation. So in a sense, you are already pressuposing a mechanistical view of the world, that is why you feel like anything but a mechanism helps/works.

    Was I clear this time?

    *is just that usually I like to the extra mile to argue for a proposition instead of just assert it... so my apologies for being most unclear*

    ReplyDelete
  69. @seanrobsville,

    "no computational model of the mind (or attempted implementation of such a model) contains within itself any intentionality"

    It seems to me that this dichotomy is founded on two assumptions. The first is that a computational model cannot possibly have intentionality of its own. The second is that my intentionality was in no way imposed on me from the outside.

    I am shocked to discover that both of these statements seem somewhat implausible to me. Is this really what you are avering? That words or equations or even objects can have no meaning of their own, without a reader? That the meaning I as a reader generate in them is so isolated from reality that it accepts no input?

    "Now if we assume that computationalism is equivalent to physicalism (Church-Turing-Deutsch principle)"

    I think it best not to assume that computationalism is equivalent to physicalism. I think there are probably several people (and a couple of sophisticated computer programs simulating people) here who don't believe it. I certainly don't anyway, so arguments founded on it will not get a fair hearing from me.

    Leaving David Deutsch out of it though : what do you think of just the Church-Turing thesis? In particular, couldn't a universal machine have intentionality of its own? Certainly, machines are routinely constructed which handle inputs not forseen in their design, and which have no meaning to the designer.

    ReplyDelete
  70. Eduardo, I think I have already allowed for the possibility in my earlier posts that my question might not be a good one. I may well have blinders on caused by my naturalism, although I am not 'committed' to naturalism in the sense that I will pull my hair and gnash my teeth if it turns out not to be true - I generally operate under the assumption that l do not have a privileged, complete or even necessarily good understanding of ultimate cosmic reality.

    But I guess I would •like• to see an explanation of, for example, the problem of consciousness, that •includes• (or leads us to) the mechanical/empirical element - it doesn't have to consist •only• of the mechanical element - from a philosophy that purports to provide us with a more comprehensive or fundamental understanding of nature. (I am not suggesting naturalism provides this.) Do or do you not think that the mechanical/empirical element could have a potential bearing on the metaphysical element of a scientific and philosophical problem such as consciousness? Or is that irrelevant •even in principle•, as it may well be?

    ReplyDelete
  71. But I guess I would •like• to see an explanation of, for example, the problem of consciousness, that •includes• (or leads us to) the mechanical/empirical element - it doesn't have to consist •only• of the mechanical element - from a philosophy that purports to provide us with a more comprehensive or fundamental understanding of nature.

    The difficulty that you're having is that you're still thinking with only two causes. If you go back to the four causes, then it all makes sense.

    Material cause: neurons, etc.

    Efficient cause: neurons bumping into each other.

    Formal cause: what might be called "emergent structure" in some circles, "holism" in others, or "form" in Aristotelianism.

    Final cause: the intentionality or aboutness pointing outward from the form.

    It isn't that Aristotelianism ignores the "mechanical" data--it's just that modern Aristotelian writers generally focus on the formal and final causes of things, because the material and efficient causes have been covered. There is no problem of consciousness for Aristotelianism. It considers every substance (form + matter hybrid) to be a holistic, emergent, irreducible pheonomenon, and so humans and their minds really are not special in this regard. The difference occurs in the total immateriality of reason, which, while complicated to explain, rests on the mind's ability to take on the forms of other things without being destroyed.

    ReplyDelete
  72. @reighley

    A universal Turing machine consists of an input/output tape which is manipulated by an arbitrarily large state transition table. It is difficult to see how a state transition table or its elements can be 'about' anything apart from the transition to the next state.

    The situation is analogous to that of the Chinese room, with the I/O tape being equivalent to the Chinese and English notes passed to and fro, and the state transition table being equivalent to the pseudo-translator's rulebook. The same arguments apply.

    ReplyDelete
  73. Anon

    Read what Rank told you I guess he sort of gives a better point.

    Although I am not saying that you have any particular bias or convictions, is just as I have said cherished premises that guides our epistemologies.

    but I do see your point.

    ReplyDelete
  74. @seanrobsville,

    "A universal Turing machine consists of an input/output tape which is manipulated by an arbitrarily large state transition table. It is difficult to see how a state transition table or its elements can be 'about' anything apart from the transition to the next state."

    I meant to place emphasis on the Universal aspect of the machine. Some Turing machines can only compute simple things (no matter how we phrase the inputs) and some may compute any computable function (if we include in the input which function we want computed).

    I imagine that such a machine in its initial state knows nothing and is directed at nothing. The state machine itself is not directed anywhere by anybody (except for the rather broad constraint that it computes computable functions). Neither is a newborn baby capable of forming thoughts "directed at" anything. This is learned, it comes in from the environment.

    So I will accept, at least for present purposes that the machine in it's start condition is not directed at anything or capable of understanding. But why shouldn't I expect it to behave like a baby and develop an understanding of the contents of the tape as it goes through the process of examining it.

    Put another way : a particular state of the machine can actually be said to derive it's meaning exclusively from the internal construction of the machine insofar as any state represents a universal : a class of inputs which would have placed the machine in that state.

    ReplyDelete
  75. Thanks for your reply, rank sophist. I didn't think Aristotelianism ignores the mechanical data, just that it doesn't lead us to it. Shouldn't it? After all, historically there was an explosion of mechanical data and in our understanding of nature once Aristotelianism was dumped, even if we grant that it was at the cost of creating problems (consciousness, etc.) that weren't there before. (One could ask, were these problems 'created' or 'discovered', and is the fact that consciousness is not also a problem for Aristotelianism actually a problem for Aristotelianism?) The 'mechanickal philosophy' did lead us to it; we did understand more of nature (if perhaps less of ultimate things). Obviously, this is one of the things that makes naturalism, for all its shortcomings, seem compelling.



    ReplyDelete
  76. Eduardo (and rank sophist),

    I do agree with your thoughts about my 'convictions' possibly leading me in the wrong direction. That's why I read this blog, of course - to explore otherwise. These things can take time....

    ReplyDelete
  77. Anon

    My personal opinion, I will try to clear about this, is that what we believe often steams from conclusion but the premises are not always clear as day light. So we ought to think about them.

    yeah I know random.

    ReplyDelete
  78. @Reighley
    "But why shouldn't I expect it to behave like a baby and develop an understanding of the contents of the tape as it goes through the process of examining it."

    Because there is nothing there that is capable of developing understanding. A tape is still a tape no matter how long it becomes. Similarly with the state transition table, which is just a set of electrical relays, no matter how numerous.

    A Turing machine is no more likely to develop an understanding of its operations than my DVD player is to develop an understanding of Shakesperean drama by playing a DVD of Hamlet.

    ReplyDelete
  79. "I didn't think Aristotelianism ignores the mechanical data, just that it doesn't lead us to it. Shouldn't it? After all, historically there was an explosion of mechanical data and in our understanding of nature once Aristotelianism was dumped, even if we grant that it was at the cost of creating problems (consciousness, etc.) that weren't there before."

    You make it sound like "Aristotelianism" was holding science back. Have you read Feser's The Last Superstition? He gives a pretty good historical overview of the metaphysical shift. Aristotle's science was... pretty bad, and was thus dumped in light of new evidence. But his metaphysics was fine, and was dumped... on a metaphysical whim I guess.

    ReplyDelete
  80. @seanrobsville,

    "Because there is nothing there that is capable of developing understanding."

    This is what is at issue. I could make a similar argument about a newborn baby. It eats, it poops, it cries. A machine that did the same would not be hard to build. A sea slug is not so different.

    Why does the baby eventually understand Hamlet, but the sea slug never does?

    Likewise the DVD player is a bad analogy. The whole point of the Church Turing thesis is that machines which can compute any computable function are the objects of interest because human beings seem to be able to compute any computable function and have trouble computing non-computable ones. DVD players are not such. In particular they have a very short memory. A human child requires a gigantic input in order to grasp even simple concepts.

    A machine might be expected to behave similarly.

    In sum : the machine by itself, like a new born baby, would not be expected to have an understanding of anything. None the less, it seems just as possible that a suitably constructed machine would show the same capacity to grow an understanding, to acquire it from the environment, as a baby does.

    It is also perfectly reasonable to suppose that not all machines would be able to do this, as not all creatures with nervous systems show the same capacity as a newborn child. So it is not a valid argument to point to a simple machine like a DVD player and generalize. The machine must at least be granted the capacity to compute any computable function an a long memory before the analogy is even remotely plausible.

    ReplyDelete
  81. "Put another way : a particular state of the machine can actually be said to derive it's meaning exclusively from the internal construction of the machine insofar as any state represents a universal : a class of inputs which would have placed the machine in that state."

    You (a subjective agent with intrinsic intentionality) are the one making the link between "derived meaning" and the "internal construction." Correct me if I'm wrong, but it seems like you are providing the machine with derived intentionality, and then calling that intrinsic intentionality.

    ReplyDelete
  82. Black Luster,

    Yes, but it wasn't only the dumping of Aristotle's science that led to the run of discoveries - early modern science was conducted under a completely different philosophy of nature - different ideas of causality, mind, etc. Yet it was immensely successful. (In that sense, the reasons •why• A.'s metaphysics were dumped are irrelevant, although to some degree the moderns did 'read' the metaphysics out of, and then back into, the new evidence.) Don't you find that curious in light of the assumption that A-T is correct? Shouldn't early modern science's adoption of an erroneous metaphysics have led to failure instead? (Perhaps that failure simply took a few centuries and is occurring now.)

    I don't know if Aristotelianism held science back (there are also contingent historical factors involved in the 'rise of modern science'), although the early moderns •thought• it did; but its elimination certainly •didn't• hold science back at all.

    I haven't yet read TLS (you may cast your stones deservedly), although I am about halfway through Burtt's 'The Metaphysical Foundations of Modern Science', referenced in the OP. It's a lot to take in - I'll have to read it again. I will read TLS eventually.

    ReplyDelete
  83. After all, historically there was an explosion of mechanical data and in our understanding of nature once Aristotelianism was dumped, even if we grant that it was at the cost of creating problems (consciousness, etc.) that weren't there before. (One could ask, were these problems 'created' or 'discovered', and is the fact that consciousness is not also a problem for Aristotelianism actually a problem for Aristotelianism?) The 'mechanickal philosophy' did lead us to it; we did understand more of nature (if perhaps less of ultimate things). Obviously, this is one of the things that makes naturalism, for all its shortcomings, seem compelling.

    Mechanical philosophy also created reductionism, determinism, the problem of induction, Hume's skepticism about causality, the grue-and-bleen paradox, the is-ought (fact-value) problem, idealism, the interaction problem, the problem of qualia and eliminative materialism, among other things. I can't deny that it led to a rapid advancement in science, but that, in itself, does not make it correct. Incorrect beliefs and actions can have positive outcomes, even in scientific practice itself. Also, given the fact-value distinction, you could not coherently say that the rapid advancement of science was even a good thing under your own system. You could merely say that it happened, without saying that it was preferable to any other course of action.

    ReplyDelete
  84. I can't deny that it led to a rapid advancement in science, but that, in itself, does not make it correct.

    I do question this. I agree there was a rapid advancement in science, but I find it odd to regard 'mechanical philosophy' as what led to it, as opposed to a way of thinking that's entirely compatible with Aristotileanism anywayw.

    ReplyDelete
  85. @reighley & anonymous
    I would propose that intentionality is either itself axiomatic, or is an aspect of an axiomatic foundation of reality that is not reducible to other axiomatic foundations, such as process/causality or structure/form.

    ReplyDelete
  86. rank sophist,

    Might those problems be good things, in the sense of problems that would have been/are inevitably necessary to deal with in order to understand nature?

    I agree that the good results of modern science don't tell us whether the mechanical philosophy is true, but I still find it curious that it led to such incredible success - on such a scale that I question whether 'Incorrect beliefs and actions can have positive outcomes' is valid (and, in any case, good/true/correct things can also have negative outcomes).

    Can we say that had the West stuck to Aristotelianism that we would have had the rise of modern science regardless? Since, under A-T, we would already know what qualia and consciousness are, perhaps we wouldn't have bothered to investigate. It seems to me that, at worst, the mechanical philosophy was an extremely fortuitous error.

    ReplyDelete
  87. Crude,

    "I do question this. I agree there was a rapid advancement in science, but I find it odd to regard 'mechanical philosophy' as what led to it, as opposed to a way of thinking that's entirely compatible with Aristotileanism anyway."

    I would hypothesize that the majority of scientists in the modern era weren't/aren't even acutely aware of their metaphysical assumptions. When they are asked about it, they read metaphysics out of the method, as mentioned in the OP.

    sean,

    "I would propose that intentionality is either itself axiomatic, or is an aspect of an axiomatic foundation of reality that is not reducible to other axiomatic foundations, such as process/causality or structure/form."

    If you are saying that intentionality can't be explained in terms of physics, I'm inclined to agree. I'd even say syntax and semantics aren't explainable in the physics of a process/system.

    ReplyDelete
  88. I do question this. I agree there was a rapid advancement in science, but I find it odd to regard 'mechanical philosophy' as what led to it, as opposed to a way of thinking that's entirely compatible with Aristotileanism anywayw.

    If you've read much about Francis Bacon's Novum Organum, it's pretty obvious that the new scientific method was explicitly based on ignoring any formal or final causes in favor of study of efficient and material causes. It grew directly out of mechanism. Bacon says that there's no knowledge to be found in studying forms--he says that true knowledge and the hope for relieving the "suffering of man" are in the parts of nature that can be measured and abused into submission. To top it off, he was an open nominalist. It was all connected: modern scientific practice and mechanism cannot be separated. Of course, it's certainly possible to adjust scientific practice so that it combines what we've learned from mechanism with the critical study of emergence. That, though, would cease to be modern scientific practice as defined by its founders and would become something else.

    ReplyDelete
  89. Crude,

    I think I basically agree with what you're saying here. It's odd to say that a four-cause system is an Alternative to a two-cause system, when it reality, the four-cause system (Aristotelianism) includes the two-cause system.

    That said, when people talk about the two-cause system, they do talk about it as if it is absolute, the end of the discussion, so in that way it is an alternative in a certain sense.

    But it does pose significant (conversation, philosophical, societal) problems when you frame it as one system versus the other. Because people see scientific advancement, and they're like, "Why would we want an alternative to This! It's clearly working out!"

    Emphasizing that the system that lead(s) to scientific advancement is subsumed by a larger, fuller metaphysics is completely vital. It's just nearly impossible to convince anyone of. They'll just keep saying, "dude, that was the dark ages, now we have computers, get out of here" and miss the point completely.

    It's always more fruitful to emphasize, as was hinted at by Rank, that the system that leads to those scientific advancements says nothing about the advancements being good or bad one way or the other. They may bite that bullet, but it's a hard (and may be even impossible) bullet to bite.

    ReplyDelete
  90. Rank,

    If you've read much about Francis Bacon's Novum Organum, it's pretty obvious that the new scientific method was explicitly based on ignoring any formal or final causes in favor of study of efficient and material causes. It grew directly out of mechanism. Bacon says that there's no knowledge to be found in studying forms--he says that true knowledge and the hope for relieving the "suffering of man" are in the parts of nature that can be measured and abused into submission. To top it off, he was an open nominalist. It was all connected: modern scientific practice and mechanism cannot be separated.

    The problem I have with what you're saying is that you're discussing a historical development, whereas I'm talking about the actual reasoning employed. To put it in perspective: you say that Bacon advocated putting aside formal and final causes to study material and efficient causes. I agree. But there's nothing about Aristotileanism that says they cannot do such a thing, even within their own system, as if saying 'I'm going to study the material and efficient causes of X, and put formal/final questions aside' is unthinkable under Aristotileanism, or that modern science (as science) contradicts Aristotileanism.

    Here's another way to put it: if we're going to talk about what the prevailing philosophical/metaphysical view was that came with science, 'Cartesian dualism' - historically - seems like the particular choice to go for. But few people seem willing to say Cartesian Dualism led to the advances of science - they beg off from that and focus on the success of a narrower focus or even methodology. But said focus and methodology isn't incompatible with Aristotileanism anyway.

    Joe K,

    That said, when people talk about the two-cause system, they do talk about it as if it is absolute, the end of the discussion, so in that way it is an alternative in a certain sense.

    But it does pose significant (conversation, philosophical, societal) problems when you frame it as one system versus the other. Because people see scientific advancement, and they're like, "Why would we want an alternative to This! It's clearly working out!"


    I may be missing something, since it seems like you're saying what I'm saying. Rather, I'm just pointing out a similar objection, but from a different perspective. I don't object to any of Rank's stated criticisms or observations - I simply question a particular identification of the method with mechanistic philosophy such that it implies the method and discoveries are somehow incompatible with Aristotileanism.

    ReplyDelete
  91. But it does pose significant (conversation, philosophical, societal) problems when you frame it as one system versus the other. Because people see scientific advancement, and they're like, "Why would we want an alternative to This! It's clearly working out!"

    Ah, pragmaticists and their lineage. Rank critiqued them really nicely in that other Nagel post.

    ReplyDelete
  92. Might those problems be good things, in the sense of problems that would have been/are inevitably necessary to deal with in order to understand nature?

    No. I don't have time to explain the details, but the grue-and-bleen paradox, the problem of induction, the impossibility of understanding causality and the contradiction of physical minds all make nature completely unknowable and render science impossible. It isn't a matter of overcoming these things in order to understand nature: if even one of them is true, then everything that we presume to know about nature is automatically false. Plus, it means that no correct beliefs can ever be formed about nature.

    I agree that the good results of modern science don't tell us whether the mechanical philosophy is true, but I still find it curious that it led to such incredible success - on such a scale that I question whether 'Incorrect beliefs and actions can have positive outcomes' is valid (and, in any case, good/true/correct things can also have negative outcomes).

    We learned a lot from Nazi human experiments. Does that in itself make them correct? Can you even say that something is correct or incorrect under mechanism?

    Can we say that had the West stuck to Aristotelianism that we would have had the rise of modern science regardless? Since, under A-T, we would already know what qualia and consciousness are, perhaps we wouldn't have bothered to investigate. It seems to me that, at worst, the mechanical philosophy was an extremely fortuitous error.

    Mechanical philosophy created modernity, which created the greatest horrors ever seen on earth. If that's what you call a "fortuitous error", then I'd hate to see what you call a serious one. Regardless, as I said, you can't coherently accept mechanism on the grounds that it helped science. Understood properly, it refutes science and invites relativism and total skepticism.

    ReplyDelete
  93. Crude,

    Oh, no, I'm not disagreeing with you at all. I likewise don't like the idea that the methods/discoveries of the scientific revolution are incompatible with Aristotelianism. I'm just trying to explain the EITHER two-causes OR four-causes phenomenon. That is, why it's framed that way. I think it's a people problem, not a philosophy one.

    ReplyDelete
  94. Joe K,

    Oh, no, I'm not disagreeing with you at all. I likewise don't like the idea that the methods/discoveries of the scientific revolution are incompatible with Aristotelianism. I'm just trying to explain the EITHER two-causes OR four-causes phenomenon. That is, why it's framed that way. I think it's a people problem, not a philosophy one.

    Alright, I think I get what you mean. If you mean by 'it's a people problem' is 'you have to find a way to communicate these ideas in a way people can grasp, and grasp easily', sure, I agree. Really, that's something I agree with - even something I think I'm addressing.

    I know it seems when I jump on these things, I'm jumping on such minor points - it's not like I misunderstand rank on this or anything. It's that the minor things, the language, the way of framing it, gets to me and I think adds to the problems. So I tend to focus on it.

    ReplyDelete
  95. @Anonymous,

    "Correct me if I'm wrong, but it seems like you are providing the machine with derived intentionality, and then calling that intrinsic intentionality."

    Consider this my attempt at correcting you, because I think you are wrong.

    Here are the things which I am imposing from without :

    I am distinguishing the "machine" from the environment. This is of course the same luxury I allow myself, to be distinguished from the environment so I can't deny it to the machine.

    I am supposing that the machine has "internal states", so I allow more than one material configuration to constitute "the machine". Again, as I do for myself.

    I am supposing that the machine is coupled to the environment so that its internal state change as a result of input (which may include, without loss of generality, simply the passage of time). This is just the usual definition of machine in this context.

    I do not require that this internal state means anything to me. In fact in the general case it will not, because deriving the class of possible input sequences from the state of the system is somewhat like reassembling a pig from sausages. Possible in principle, but not in practice.

    Since I don't understand where the machines state comes from or its consequences, I don't think I can possibly be the source of whatever meaning the machine has in it (if meaning it has).

    What I am proposing is that this internal state of the machine _has meaning to the machine_. Because it includes anything that the machine "remembers" about what happened to it and embodies all of the consequences of its "knowledge" and its responses to future inputs.

    It seems to me that this internal state must mean something (as it is correlated to real facts about the world, and has practical consequences). On the other hand, in the usual case I posses no way of decoding or making sense of the state of the machine except possibly by allowing the machine to run and hope that it tells me what I want to know in an output.

    So it seems to me that the meaning is there, but that I don't know what it is. If you will forgive the notion of meaning I have used here, which is to say that the information bares some relationship to facts, and has practical consequences.

    @seanrobsville

    "I would propose that intentionality is either itself axiomatic, or is an aspect of an axiomatic foundation of reality that is not reducible to other axiomatic foundations, such as process/causality or structure/form."

    I am certainly willing to assume that this is so, in the absence of an actual reduction. But just because intentionality is its own thing, doesn't mean that a pile of electrical relays cannot have any. Or for that matter that it is independent of cause and form. I would not say that cause and form are independent of one another. "Irreducible to" does not mean "unrelated".

    ReplyDelete
  96. I think this is relevant. Pruss briefly argues that belief in uncaused events logically leads to skepticism about pretty much everything.

    http://alexanderpruss.blogspot.com/2012/11/scepticism-and-causeless-events.html

    ReplyDelete
  97. I don't think it's the case that it was merely a two-cause versus four-cause system, and that four-cause system 'contains' the two-cause - or, at least, that is too simple. The fundamental categories were different: mass, space, time, force, etc.; the idea that space and matter were the same thing; the idea that qualities were just quantities (mathematics is •true• reality); the idea of 'simplicity', etc. All this was from and/or resulted in the new metaphysics.

    ReplyDelete
  98. "I am certainly willing to assume that this is so, in the absence of an actual reduction. But just because intentionality is its own thing, doesn't mean that a pile of electrical relays cannot have any. Or for that matter that it is independent of cause and form. I would not say that cause and form are independent of one another. "Irreducible to" does not mean "unrelated"."

    I kinda agree. I see no reason why a silicon brain cannot ever have intrinsic intentionality. However, I think that such intrinsic intentionality would be inexplicable using just material and efficient causes.

    ReplyDelete
  99. Anon,

    I don't think it's the case that it was merely a two-cause versus four-cause system, and that four-cause system 'contains' the two-cause - or, at least, that is too simple.

    I don't think anyone is saying that, say, 'materialism is just Aristotileanism without some additional stuff!' Only that the methods and advances of science are entirely compatible with Aristotileanism - trivially so once you consider that science isn't metaphysics anyway.

    ReplyDelete
  100. Crude,

    Completely legitimate thing to focus on! The abortion wars are lost much of the time because of the way the debate is framed.

    ReplyDelete
  101. Too many Anonymouses.

    ReplyDelete
  102. Joe K,

    Completely legitimate thing to focus on! The abortion wars are lost much of the time because of the way the debate is framed.

    Exactly. Man, I am so tempted to comment additionally about *why*, but for the sake of this conversation, I'm going to hold off and leave it at my agreement.

    ReplyDelete
  103. But there's nothing about Aristotileanism that says they cannot do such a thing, even within their own system, as if saying 'I'm going to study the material and efficient causes of X, and put formal/final questions aside' is unthinkable under Aristotileanism, or that modern science (as science) contradicts Aristotileanism.

    Well, I don't want to get into another big argument, but it is true that modern science is incompatible with Aristotelianism to an extent. Modern science as defined by Bacon and company is necessarily reductionistic and nominalistic. If you take out reductionism and nominalism, then what you're left with is not modern science anymore. It's certainly possible for an Aristotelian to focus on efficient and material causes (and therefore engage in mechanistic modern science), but Aristotelian science will always contain studies of formal and final causes--things ruled out from the beginning by modern science. This is not to say that modern scientists don't smuggle talk of forms and teleology in through the back door, nor is it to say that Aristotelianism can't absorb most of mechanistic science. My point is that a defining characteristic of modern science is the eschewal of formal and final causes, and that any science that takes those things into consideration is no longer modern science, but is rather something new. To say otherwise is a bit like copying the materialists in their constant redefinition of just what "materialism" means.

    Regardless, I think it's a waste of time to argue about this, so let's just drop it. Even if you respond, I'm not going to follow up--I just wanted to clarify my position. Sorry.

    ReplyDelete
  104. rank sophist,

    I'm not really arguing •for• mechanism against A-T in the sense that I'm trying to 'defeat' A-T or have some 'gotcha' moment. Those things are well beyond my meager capacity. I do have the typical modern sense, rightly or wrongly, that Aristotelian explanations of, for example, consciousness are 'empty' or devoid of meaningful content. What you quoted from Bacon about there being no meaningful knowledge in formal and final causes seems right to me. But again, I'm really just trying to learn about A-T, which I find impressive and formidable, so I am trying to find out what meaning, if any, there is to the fact that A-T, a philosophy of nature, didn't lead to the modern discoveries of the workings of nature, and the mechanical philosophy, egregiously wrong under the A-T view, did.

    In any case, thanks (everyone) for your replies.

    ReplyDelete
  105. Rank,

    Modern science as defined by Bacon and company is necessarily reductionistic and nominalistic. If you take out reductionism and nominalism, then what you're left with is not modern science anymore.

    I'll simply say that I don't believe modern science requires a commitment to reductionism or nominalism in the relevant senses, and that once you recognize that the whole thing integrates with Aristotilean metaphysics anyway. I never meant to imply that you can just have modern science and 'That's Aristotileanism!' - it's merely a part, a component, of a larger (metaphysical, philosophical) project.

    No problem on the lack of response. I think it's relevant, but not every exchange needs to lead to some big ongoing conversation.

    ReplyDelete
  106. As the internet kids might say, "Well, that de-escalated quickly." So, what are your plans for this fine Sunday night, Crude? Anything cool? I'm supposed to be studying for a final right now, and this Was the perfect distraction, but now I just have more studying.

    ReplyDelete
  107. Crude said:

    "...Only that the methods and advances of science are entirely compatible with Aristotileanism - trivially so once you consider that science isn't metaphysics anyway."

    Science isn't metaphysics, but early modern science was most definitely pursued under a metaphysics, which (deliberately so) wasn't Aristotelianism.

    If we look at, say, modern biology, specifically the idea of 'natural selection' - whatever you may think of its prominence or not in adaptation, it has at least •some• role - could natural selection have been discovered under Aristotelianism?

    (I apologize if this is all tedious - I know I'm largely repeating myself. I'll quit soon.)

    ReplyDelete
  108. Similarly, what materialistic explanations do is to capture those aspects of the natural world susceptible of a materialist analysis -- breaking down larger systems into component material parts, mathematically modeling the parts and their combinations, testing the predictions that follow from these models, and so forth.

    This is where I disagree with you Edward. Maybe you're being to kind to the materialists? I really don't know.

    But even those so-called "materialistic explanations" are not devoid of higher-level concepts/forms/holistic systems/etc. The break down of these things into smaller parts is still not devoid of things that the materialist account would require. Furthermore, what on earth does mathematics have to do with materialism? I was reading an article recently by a Theist, who criticized this reliance of naturalism on mathematics and went on to describe mathematics as a trojan horse in the citadel of naturalism. I think he was spot on. In short, the higher-level structures, formal causes and so on are simply not made explicit in the language used to describe whatever scientific fact the researcher allegedly describes. That's all it is. It is nonetheless implied not only in the results but also in the method itself.

    You're too generous to the materialists sometimes. ;-)

    ReplyDelete
  109. Joe K,

    Just getting some work done, eventually. Nothing major here.

    Anon,

    Science isn't metaphysics, but early modern science was most definitely pursued under a metaphysics, which (deliberately so) wasn't Aristotelianism.

    If we look at, say, modern biology, specifically the idea of 'natural selection' - whatever you may think of its prominence or not in adaptation, it has at least •some• role - could natural selection have been discovered under Aristotelianism?


    To the natural selection question, yes - you'll even see that cited in The Last Superstition as yet another example of a concept that fits just great with Aristotileanism. I think that especially goes if you put 'its prominence or not in adaptation' aside, since if you do that it's just a minor retooling of animal husbandry.

    You say that 'early modern science' was pursued under a metaphysic that wasn't Aristotileanism. Sure, let's grant that now (though we can always talk about what does or doesn't get smuggled into science.) But the relevant parts of modern science, I maintain, are trivially compatible with Aristotileanism anyway.

    Imagine of Roger Bacon said, 'formal and final causes are supremely important - but I think more work should be done focusing on material and efficient causes exclusively, so that's what my research program will focus on, while others in other 'fields' will focus on formal/final causes'. Will there be any contradiction? Any important parts of science left out, or barriers erected?

    I think the answer is clearly no.

    ReplyDelete
  110. Anon: How does Aristotelianism explain how, for example, neurons produce consciousness?

    I think what Eduardo was getting at in his posts (forgive me Eduardo if I misstate your views!) is that your question itself shows a bias in that it assumes that neurons produce consciousness.

    I'm no expert, but for an A-T learner (such as myself): neurons and consciousness do not 'cause' each other but rather are two parts of a holistic organism (the 'human form').

    An A-T informed scientist can study how neurons work and how they influence consciousness (and vice-versa) without being confined to an 'X causes Y' paradigm.

    ReplyDelete
  111. "I'm supposed to be studying for a final right now, and this Was the perfect distraction, but now I just have more studying."

    This is my exact situation as well. If I wanted to distract myself "productively" I'd probably be reading actual literature like Feser's or Pruss's instead of hitting F5 every few moments in a combox. But its too entertaining.

    ReplyDelete
  112. Anon,

    It's perfectly fine for you to keep posting on this issue. In general, the regulars here enjoy debating naturalists who have a legitimate interest in Aristotle and Aquinas. The ire is directed at cranks and trolls--neither label fits you.

    But again, I'm really just trying to learn about A-T, which I find impressive and formidable, so I am trying to find out what meaning, if any, there is to the fact that A-T, a philosophy of nature, didn't lead to the modern discoveries of the workings of nature, and the mechanical philosophy, egregiously wrong under the A-T view, did.

    One big reason for this is that Aristotle himself was a scientist, and the scientific practice that he propounded in the Organon was lousy. He dismissed experiment (a critical part of the big advancements made by early-Islamic science and modern science) and hypothesis in favor of purely deductive science. From this, we were led to absurdities like the four elements. Plus, his own scientific beliefs (regardless of his practices) were way off--another thing that was passed down. Had early Islamic science been transmitted to Europe, as the Aristotelian philosophy of Averroes and Avicenna was, then I can only assume that things would have been different. As it was, Aristotle's philosophy was tangled up his science to such an extent that the two seemed inseparable.

    I should add that your belief that forms are irrelevant is due to your misunderstanding of what a form is. This is very common--I myself was baffled by them for quite awhile. But you've got to realize that a form is nothing fancier than water having the chemical formula H2O. It's not "spooky" or "extraneous": it's just the common sense view. Again, though, I'm too busy to get way into this topic. If I had more time, I'd give you the full breakdown. As it is, I can only recommend that you read TLS while ignoring the political parts. (All apologies to Prof. Feser, but the political element to TLS has prevented me from recommending it to people. The average liberal atheist will be put off by the first page alone. Aquinas is far superior in this regard, but it doesn't tackle the same subject matter.)

    ReplyDelete
  113. rank sophist,

    Thanks, I didn't want to seem a troll - my interest is genuine, even if my knowledge is poor and my confusion is great. Yes, the 'tone' was originally the reason I didn't order TLS, although I have read this blog long enough and accept Dr. Feser's explanations for that tone, so that's really no longer a good excuse. (Although as a liberal atheist, perhaps I will still be mortified nonetheless.) I do wish he would write a complete, all-in-one philosophy of nature book.

    Daniel Smith,

    My question, though stated as such, doesn't really depend on whether or not neurons 'produce' consciousness, only that there is a neuronal 'role' in consciousness, and that that role was/is being discovered under a non-Aristotelian paradigm.

    ReplyDelete
  114. Some excerpts from Francis Bacon's Novum Organum (which excerpts, as will be obvious to the reader, are cherry picked):

    o {I]n the true course of experiment, and in extending it to new effects, we should imitate the Divine foresight and order; for God on the first day only created light, and assigned a whole day to that work without creating any material substance thereon. In like manner we must first, by every kind of experiment, elicit the discovery of causes and true axioms, and seek for experiments which may afford light rather than profit.

    o [A]ny one who properly considers the subject will find natural philosophy to be, after the Word of God, the surest remedy against superstition, and the most approved support of faith. She is, therefore, rightly bestowed upon religion as a most faithful attendant, for the one exhibits the will and the other the power of God. Nor was he wrong who observed, "Ye err, not knowing the Scriptures and the power of God," thus uniting in one bond the revelation of his will and the contemplation of his power.

    o Let us begin from God, and show that our pursuit from its exceeding goodness clearly proceeds from him, the author of good and father of light. Now, in all divine works the smallest beginnings lead assuredly to some result, and the remark in spiritual matters that "the kingdom of God cometh without observation," is also found to be true in every great work of Divine Providence, so that everything glides quietly on without confusion or noise, and the matter is achieved before men either think or perceive that it is commenced.

    o [I]t is worthy of remark in Solomon, that while he flourished in the possession of his empire, in wealth, in the magnificence of his works, in his court, his household, his fleet, the splendor of his name, and the most unbounded admiration of mankind, he still placed his glory in none of these, but declared that it is the glory of God to conceal a thing, but the glory of a king to search it out.

    o [L]et none be alarmed at the objection of the arts and sciences becoming depraved to malevolent or luxurious purposes and the like, for the same can be said of every worldly good; talent, courage, strength, beauty, riches, light itself, and the rest. Only let mankind regain their rights over nature, assigned to them by the gift of God, and obtain that power, whose exercise will be governed by right reason and true religion.

    o [I]t is only for God (the bestower and creator of forms), and perhaps for angels and intelligences, at once to recognize forms affirmatively at the first glance of contemplation: man, at lest, is unable to do so, and is only allowed to proceed first by negatives, and then to conclude with affirmatives, after every species of exclusion.

    ReplyDelete
  115. rank sophist,

    Your remark about Aristotle's dismissal of experiment is interesting. I wasn't aware of that. I know he dissected animals and was basically the first biologist, so that surprises me somewhat. That would be a key failure given the critical importance of experiment to the success of science, but one that wouldn't affect the validity or truth of his metaphysics.

    ReplyDelete
  116. Glenn, that last one makes it seem like Bacon said that humans are incapable of fully comprehending forms. If that's what he meant, then I guess it's understandable why he didn't pursue them.

    ReplyDelete
  117. If the Bacon quotes are directed at me, I am aware that he and all the early modern scientists were religious, and that God was, for them, the reason behind the order in nature.

    ReplyDelete
  118. Black Luster,

    Sounds that way to me, too.

    OTOH, an alternative explanation may be that he just really, really wanted to dive into efficient and material causes, and so fashioned a rationalization which favored his strong desire.

    Although people have been known to do such a thing (characterize one thing in such a way that they might be better justified in persuing another thing), I've no idea if there is any genuine truth or credibility to this suggested 'alternative explanation' re Bacon and forms. 'tis off the cuff speculation, is all it is.

    ReplyDelete
  119. Anonymous,

    No, it wasn't directed at you--or anyone else.

    It was just a general "Let us not forget..." kind of thing, is all.

    And that it may lend some support to Crude's "Imagine if Roger Bacon said..." is purely coincidental.

    (Although, hmm, in light of the third quote, perhaps... nah, couldn't be, right?)

    ReplyDelete
  120. Here's another aphorism from Bacon's NO, one which may admit of a modern application,

    o Some men become attached to particular sciences and contemplations, either from supposing themselves the authors and inventors of them, or from having bestowed the greatest pains upon such subjects, and thus become most habituated to them. If men of this description apply themselves to philosophy and contemplations of a universal nature, they wrest and corrupt them by their preconceived fancies[.]

    (Alas, Bacon's statement doesn't end precisely there; it goes on, and Aristotle gets caught up in the sweep.)

    ReplyDelete
  121. @Rank

    This is the first time I hear that Aristotle rejected experimentation. I know that he was interested in biology and played around with animal corpses but I was certainly not aware that he explicitly eschewed experiment.

    Do you have a direct quote by any chance? (Not calling you a liar, it's just that the claim is rather significant and would like to both see it and also see it in context)

    If you have any direct links/quotes/etc I'd like to read them. If not, no worries.

    ReplyDelete
  122. @Glenn

    (Alas, Bacon's statement doesn't end precisely there; it goes on, and Aristotle gets caught up in the sweep.)

    I'd bet that eventually Bacon gets swept in it as well as he too held certain convictions that restricted his thoughts and methods.

    ReplyDelete
  123. I have yet to meet a person who was not to some extent blinded by their own intellectual habits and pet theories.

    ReplyDelete
  124. Prof. your awesome. After I finish my degree in English I'm moving to California to pursue a career in film production. If I can find the time I'd like to take some of your classes and join the conversation. I'm in a post-modern lit. class right now with a teacher who's ready to write anyone off that isn't an agnostic/atheist liberal as an uninformed simpleton. It frustrates me that I don't know the Aristotelian/Thomist position well enough to defend it. You had an earlier post that references how students are brow beaten into agreement with current academias point of view. I'm not sure where I lie politically but in my experience most conservatives (with the obvious exception of yourself and I'm sure many others) have inherhited values that they can't support (not that their values are right or wrong 2+2=4 whether someone tells you it or you do the math) and the stick to it regardless of counter-arguments. As annoying as that is I can honestly live with it. The thing I find infuriating with most liberals is that they're as close-minded the "close-minded" conservatives they thumb their noses at. Now I'm really echoing that post you made but they act like they're open to follow the argument and listen to your side but anytime I begin to open the conversation to God and to God in a classical theistic sense I feel walls go up and hackles come out. That I'm written off as some Jesus freak subversive who bases everything off of the Bible. I have to nod in complacent agreeance that any of the post-modern crap I have to read isn't BS...Well that turned into a rant. Keep writing professor hopefully I'll see you in class someday.

    In reference to Papalinga and other trolls "spare them the poisonous barbs" WWJD

    ReplyDelete
  125. "I have yet to meet a person who was not to some extent blinded by their own intellectual habits and pet theories."

    Aren't we all Reighley XD?

    ReplyDelete
  126. My professor said once that for Aristotle true science was about observing the phenomena.

    The idea to create multiple experiments and an over-arching theory for them all seems to be a very modern conclusion, especially after our success in marry multiple fields under a UNIFYING theory.

    ReplyDelete
  127. Daniel Smith

    Yeah the point was that.

    Not necessarily eliminating the possibility of neurons having a relation to consciousness but the question was somehow limting the answer while being asked.



    ReplyDelete
  128. Eduardo,

    Just to stick up for Aristotle in part - I think one could fairly say that what he was personally interested in in general was the sort of thing where observing the phenomena was the most appropriate response.

    Not engaging in experiments was a setback, but we observe to learn even now - particularly with biology, etc. People who want to learn about animals put a lot of emphasis on watching them in their natural habitat, for example, not just throwing them into a room and experimenting on them.

    ReplyDelete
  129. I think the problem, or rather my theory, is that we became more pragmatic through the years and slowly we saw how important it was to do THOSE EXTRA experiments.

    Back in Aristotle day there was probably nothing close to pragmatism, nobody could have thought the to do experiment would spring things like engineering.

    It was different times that is all.

    Who knows maybe in the future we will be just simpletons that haven't noticed A, B and C and what marvelous things they were XD.

    But on to sort of saving Aristotle, I personally like METHODS, even old ones, so to me what Aristotle and Co, have created as methods are just as important as Bacon, Newton, Descartes have created as methods and of course just as important as the man and woman creating method today to analyse and discuss data of all sorts.

    That is what I am usually in love about the Sciences, THE METHODS!

    Of course... your PBS love for science dies pretty quickly when you think like me... and it dies even more when you actually study Science. Sorry just disappointed with certain unrelated things.

    ReplyDelete
  130. That may pose a problem for dualist theories of the mind. From the article:

    "Spaun's cognition and behaviour is very basic, but it can learn patterns it has never seen before and use that knowledge to figure out the best answer to a question. "So it does learn," says Eliasmith."



    The problem with the vitual brain is that it can only mimic some of the functions a brain can do, through the imput of programmers.

    The article itself says:
    "His Waterloo team took a different approach, using computers to simulate what goes on inside the brain, similar to the way aircraft simulators mimic flight."

    So the virtual brain is nothing like a human mind (or any mind) at all, just like a Flight simulator is nothing like an airplane, but just emulates part of the flying experience.

    Also:

    "His team reports that the virtual brain can perform eight tasks that involve recognizing, remembering and writing down numbers."

    Impressive, but that hardly puts dualism in difficulty.

    Dualist (certainly not hylomorphic dualists) will not argue that such functions are immaterial.

    recognition, memory, acting are in themselves material actions

    What is immaterial is not 'recognizing' a symbol, but UNDERSTANDING what that symbol means.

    The virtual brain has a 'virtual' understanding, not a real one. It's the scientists who understand not the machine.


    "That seems to me that the computer program is displaying proto-intentionality and that it can do some proto-reasoning"

    Not really. That virtual brain can only do what it's told, as all machines do.

    Sure it can 'learn' from its environments in the sense that it can adapt in order to do what it is told more efficiently...

    ... but that is not proto-intentionality, nor 'resoning': it's computing.

    The reasoning and intentionality exist only in the software programming and hardware building by the scientists.

    This virtual brain is certainly and impressive experiment, but comes nowhere near human (or even mammal) consciousness.

    ReplyDelete
  131. @anonymous

    That virtual brain can only do what it's told, as all machines do.

    "...the process of writing algorithms does not in itself appear to be algorithmic. The real test of computationalism would be to produce a general purpose algorithm-writing algorithm. A convincing example would be an algorithm that could simulate the mind of a programmer sufficiently to be able to write algorithms to perform such disparate activities as controlling an automatic train, regulating a distillation column, and optimising traffic flows through interlinked sets of lights.

    According to the computationalist view, this 'Mother of all Algorithms' must exist as an algorithm in the programmer's brain, though why and how such a thing evolved is rather difficult to imagine. It would certainly have conferred no selective advantage to our ancestors until the present generation (even so, do programmers outreproduce normal people?).

    The proof of computationalism would be to program the Mother of all Algorithms on a computer. At present no one has the slightest clue of how to even start to go about producing such a thing.

    According to Buddhist philosophy, this is hardly surprising, as the Mother of all Algorithms is itself NOT an algorithm and never could be programmed. The mother of all algorithms is the formless mind projecting meaning onto its objects (i.e. conceptually designating meaning on to the sequential and structural components of the algorithm as it is being written).

    The non-algorithmic dimension of mind, of understanding of meaning, is needed to turn the user's (semantically expressed) requirements into the purely syntactic structural and causal relationships of the algorithmic flowchart or code.

    The computer analogy of conceptual dependency, as far as one is possible, would be the 'meaning' of symbolic variables which gets stripped out of high level languages during compilation to machine code. This removal of meaning is inevitable because a machine cannot understand, interpret, use or manipulate meaning. Only minds can grasp meaning, hence the programmer's lament:


    I'm sick and tired of this machine
    I think I'm going to sell it
    It never does do what I mean
    But only what I tell it


    - http://rational-buddhism.blogspot.com/2012/01/algorithmic-compression-and-three-modes.html

    ReplyDelete
  132. @Sean

    Can you elaborate on why the act of writing algorithms is itself not an algorithm (or at least cannot be represented by an algorithm)?

    I'm curious.

    ReplyDelete
  133. @ Anonymous
    Because algorithms (at least those that do anything useful in the real world)start life as user requirements specifications which are always about something.

    The art of writing algorithms is to understand this 'aboutness' in great detail and convert it to a set of procedures acting on the input and outputs of the system.

    The algorithms themselves are however stripped of any aboutness, and at machine-level consist of a small repertoire of aproximately 20 bit-manipulating instructions acting upon a long one-dimensional array of bits.

    Algorithms do not contain aboutness, and it is difficult to see how they could begin to understand enough about any semantically-specified system to turn it into a useful program.

    ReplyDelete
  134. Anonymous:

    You can read about Aristotle’s denunciation of experimentation in David Lindberg’s The Beginnings of Western Science on page 51, which you can read online on Google Books. He cites Aristotle’s Politics as support for this: “But then we must look for the intentions of nature in things which retain their nature, and not in things which are corrupted.” For Aristotle, “corruption” occurs when something is prevented from actualizing its final ends, which presumably he thought would occur outside of its natural environment.

    ReplyDelete
  135. yeah apprently that was the major reason to not use experiments outside the ones you found in nature already XD.

    In a sense it sort of makes sense. But had he emphasized on essence maybe he would have thought otherwise... maybe...

    ReplyDelete
  136. Actually this is quite interesting, if back then people were to expand Aristotle concept of how Science should be conducted maybe today, we would have Natural Science and Corrupted Natural Science XD!

    Man one can only wonder XD.

    ReplyDelete
  137. Related things go together, so--1. question; 2. response; and, 3. brief elaboration:

    1. Anonymous [@December 2, 2012 6:46 PM] said ...

    This is the first time I hear that Aristotle rejected experimentation. I know that he was interested in biology and played around with animal corpses but I was certainly not aware that he explicitly eschewed experiment...

    If you have any direct links/quotes/etc I'd like to read them...



    2. dguller [@ December 3, 2012 9:56 AM] said...

    Anonymous:
    You can read about Aristotle’s denunciation of experimentation in David Lindberg’s The Beginnings of Western Science on page 51, which you can read online on Google Books. He cites Aristotle’s Politics as support for this: “But then we must look for the intentions of nature in things which retain their nature, and not in things which are corrupted.” For Aristotle, “corruption” occurs when something is prevented from actualizing its final ends, which presumably he thought would occur outside of its natural environment.



    3. David Lindberg (in 1992 BWS, pp 52-53; dguller's p 51 refers to the 2008 BWS (in which some of the wording differs)):

    The world we inhabit is an orderly one, in which things generally behave in predictable ways, Aristotle argued, because every natural object has a "nature"--an attribute (associated primarily with form [which ought not be confused with its shape]) that makes the object behave in its customary fashion, provided no insurmountable obstacle intervenes... To this general statement of Aristotle's theory of "nature," we need only add two reminders: that it does not apply to objects produced artificially, for such objects possess no inner source of change, but are merely the recipients of external influence; and that the nature of a complex organism does not result from a summation or mixture of the natures of the constituent materials, but is a unique nature characteristic of that organism as a unified whole.

    With this theory of nature in mind, we can understand a feature of Aristotle's scientific practice that has puzzled and distressed modern commentators and critics--namely, the absence from his work of anything resembling controlled experimentation. Unfortunately, such criticism overlooks Aristotle's aims--aims that drastically limited his methodological options. If, as Aristotle believed, the nature of a thing is to be discovered through the behavior of that thing in its natural, unfettered state, then artificial constraints will merely interfere. If, despite interference, the object behaves in its customary fashion, we have troubled ourselves for no purpose. If we set up conditions that prevent the nature of an object from revealing itself, all we have learned is that it can be interfered with to the point of remaining concealed. Experiment reveals nothing about natures that we cannot learn better in some other way. Aristotle's scientific practice is not to be explained, therefore, as a result of stupidity or deficiency on his part--failure to perceive an obvious procedural improvement--but as a method compatible with the world as he perceived it and well suited to the questions that interested him. Experimental science emerged not when, at long last, the human race produced somebody clever enough to perceive that artificial conditions would assist in the exploration of nature, but when natural philosophers began asking questions to which such a procedure promised to offer answers.

    ReplyDelete
  138. Thanks for providing those quotes, guys. I just came here to respond to the Anons, but you beat me to it.

    ReplyDelete
  139. @Sean

    So basically we're back to the problem of intentionality then. I didn't make the connection between intentionality + algorithm. I was thinking of it in a different way. Thanks for the clarification.

    ReplyDelete
  140. @Glenn

    Thanks for the excerpt.

    I think it makes sense why Aristotle had no interest in experimentation as Lindberg explains it, given what he was interested in discovering.

    It's the good ol' axiom of research, "you get what you measure" and since his questions were not of strict scientific nature (as it would be defined today) experimentation seemed superfluous to his endeavors.

    Observing nature in it's 'natual' order I think is an integral part of understanding reality. In other words I don't think we should not limit ourselves only to artificial experimentation. I think one of the major flaws of scientism is precisely this.

    ReplyDelete
  141. I don't think we should not limit ourselves only to artificial experimentation. I think one of the major flaws of scientism is precisely this.

    You are in a state of concurrence with many people here.

    (Re the excerpt, thanks to dguller for pointing to it; I'd not have been aware of had he not.)

    ReplyDelete
  142. I agree that natural observations have merit, but sometimes there's just too much "background noise." A lab setting allows you to minimize the background effects and set up controls.

    Hey Rank, maybe it's just me (I used to lurk for a while back) but you seem to post less in comboxes than you used to. I suppose you stick to the actual literature instead?

    ReplyDelete
  143. Anonymous:

    Observing nature in it's 'natual' order I think is an integral part of understanding reality. In other words I don't think we should not limit ourselves only to artificial experimentation. I think one of the major flaws of scientism is precisely this.

    I don’t think anyone endorsing scientism would say that only artificial experimentation is appropriate to understand the natural world. There is an important role for observation and experimentation both under the rubric of scientism.

    Just saying.

    ReplyDelete
  144. Black Luster,

    It's not just you: I've definitely been around less. It isn't because I've been hitting the books more, though. After the marathon debates I was having there for a few months, I kind of burned out--and I've been focusing on things other than Thomistic philosophy, lately. All of that arguing helped me wrap my head around the key concepts. Basically, to get personal for a second, Thomism (as explained by Prof. Feser) helped me turn my life around, and now I'm focusing more on living it. I had pretty much been convinced by the ubiquitous scientism, naturalism and atheism that Christianity was false. The fact that most logical cases for Christianity were paper-thin only made me more sure that it was all outdated superstition. I came across this blog a little over a year ago, around the time that I was at my lowest point. After reading for months and months, I started to understand that what Prof. Feser was saying was correct. It was only well after this that I started posting, once I had a firm enough grasp on philosophy and logic that I felt somewhat confident in a debate.

    But, yeah; I understand this stuff well enough now that I feel secure in following it, and it's totally changed my life. I can't thank Prof. Feser enough. The flipside is that I feel less need to actually post now, so you probably aren't going to see me as often. I'll continue to drop by, though.

    ReplyDelete
  145. Rank,

    That's great to hear. I still have my doubts, but I think Feser's case is solid. There was a point where I was essentially a nihilist, but then I discovered WLC's work. From there I started roaming the blogosphere, and I found Pruss, Reppert, Feser, Vallicella and others. They shaped my outlook in life, but I still have to read the rest of their stuff. Probably during winter break.

    ReplyDelete
  146. For what it's worth, while I was a theist prior to reading TLS and Feser's work generally, what that book did manage to do was put a lot of the details about philosophy and metaphysics in stark relief, and in a way I could grasp them. I think the introduction to Thomism and Natural Law is important, but almost as important is the way it made it easy to understand the fundamental physical issues, the distinction between science and metaphysics, the intellectual commitments involved IN science and reasoning generally, etc. All that plus an understanding of why philosophy took the turns it did throughout history, or at least a good argument to that end.

    Once you understand those basic things, a whole new intellectual world opens up.

    ReplyDelete
  147. I felt the same way Crude. TLS was an introduction, though I probably need to go through it again slowly and take notes or something, check out the references, etc.

    While I was looking through WLC's work, one thing that held me back from engaging in the stuff was the ID baggage that seemed to come along with it (from my point of view, anyway). I just couldn't buy ID. The guys almost have my sympathy, but I think its a sinking ship. I'm just relieved that Feser's stuff doesn't need to go that route.

    ReplyDelete
  148. @seanrobsville,
    "According to the computationalist view, this 'Mother of all Algorithms' must exist as an algorithm in the programmer's brain, though why and how such a thing evolved is rather difficult to imagine."

    I feel quite confident of the truth of certain propositions.

    (1) this 'Mother of all Algorithms' can not possibly exist. I think it reduces to the halting problem.

    (2) in spite of proposition (1), computationalism is not disproved because in fact there is no evidence that human beings can produce a program for any specification even if such a program exists. We produce only a narrow selection of programs to do a narrow selection of things (and even then they have bugs).

    (3) programs can be constructed to generate certain classes of algorithms. I believe this because they can be constructed to prove certain classes of theorems, and because of the Curry-Howard isomorphism.

    ReplyDelete
  149. For what it's worth, I have a somewhat similar story to both Rank and Crude. I wonder how close in age we are? I wonder if age is significant to that opening up. I'm 27. And male. (I don't seem many women around here...)

    ReplyDelete
  150. It is natural enough to wonder why such flawed metaphysics could undergird such successful science, but we want to avoid falling into the trap of politicians who take credit for the results of work laid by their predecessors. We must first ask, why did the scientific revolution happen where and when it did, instead of in some time or place unhampered by Aristotelian metaphysics? If Aristotle is so bad for science, why was it when Aristotelian-based philosophy reached its highest point that the seed of modern science was planted? Modern Science is an ungrateful child who complains about his parents despite their vast influence on his future success. Of course, there are lots of convoluted historical threads with which the sound-bite summary can't be bothered. Every revolution needs a villain, and Aristotle was the obvious choice. Sure, he got a lot of things wrong compared to Newton, but then Newton got things wrong compared to Einstein, and Einstein had only a few centuries on Newton, while Newton had a couple of millennia to improve on Aristotle.

    Anyway, all that railing against the establishment didn't stop Modern Science from spending daddy's money. It always talked about rejecting formal and final causes, but as article after articles on this site makes clear, you can't have basic physics without both. Of course, the "teleology" that was rejected was in large part stuff like "polar bears have white fur so they can hide in the snow", which doesn't help you breed genetically-altered bears to sell designer furs. So that level of biological teleology was rejected in favour of chemical and physical teleology which obviously doesn't dispense with finality in any way, but merely shows more interest in one kind than in another. That's why Darwin ended up in his finch-stocked laboratory, doing beak-manipulation experiements. —oh, wait... but at least he showed you can do biological-level science without biological-level teleology —oh, wait... maybe you can't even avoid talking about it after all.

    So it's not just that Aristotelian-type philosophy "vs." science is a false dichotomy; it isn't even really Aristotelian-type science vs. modern science — the very foundation for science, that everything from a proton to a platypus has certain characteristics and behaves a certain way, just IS formal and final causality. In fact, the main success of this alleged rejection was probably just the advertising effect: you know, making your product sound great and different from all the rest, whether it is or not. It gets folks all excited and talking about your big new project. Much more important to the actual success of science was the Judeo-Christian understanding of the universe as deliberately created by a Rational Being. The world wasn't random, or part order + part chaos. Man was deliberately created as a rational being, and the world to fit him, and he had some degree of dominion over it. If the heavens could be fathomed mathematically, then there's no reason the terrestrial sphere couldn't be as well. Of course, that required more mathematics, like Cartesian co-ordinates and calculus.

    (continued...)

    ReplyDelete
  151. But if real science is so Aristotley, then shouldn't this opposition have hurt it? Well, it did, but thanks to "secretly" retaining forms and teleology all along, and because of lots of other stuff going on, it still managed to be highly successful. Most of the ill effects spilled off to the side, worming their way into the general philosophical zeitgeist — reductionism and materialism and all that. I already touched on the problems in biology, with confusion over how to explain obviously directed behaviour and Intelligent Design and trying to reduce it all to physics. And physics itself has had problems, like the indomitable billiard-ballism that plagues QM. Physics itself marched on, of course, because the billiard balls only showed up in people's philosophical interpretations, not in the equations. No matter how flawed your metaphysics, if you "just shut up and calculate", then the formulas (FORM-ulas, get it?) will carry you through. But you'll end up finding it all much "spookier" than Aristotle would have. And of course, we got all these wacky problems concerning the mind: like the "hard problem" of how to get intentionality out of non-intentioal parts, instead of the easy problem of how to get intentionality out of intentional parts.

    Of course, this is all itself a very caricatured and over-simplified view of a vastly complex series of historical events, as well as rehashing points already made by Crude, Eduardo, Rank Sophist, et alia. The main point is to help indicate why science didn't succeed simply "in spite of" bad metaphysics.

    ReplyDelete
  152. (FORM-ulas, get it?)

    How about form-ULAs?

    ULA (User License Agreement):

    1. You may use the Aristotelian notion of forms without realizing that you are.

    2. You may also employ Aristotelian logic when using the Aristotelian notion of forms, likewise without realizing that you are.

    3. You may also make fun of, sneer at, mock, disparage and otherwise show and express contempt for the Aristotelian notion of forms, as well as Aristotelian logic--they have both been around for longer than you have, therefore are more mature, and thus quite capable of tolerating (i.e., enduring) the irrational exuberance of relative youth; beside, in the final analysis you really can't do your work without 'em.

    ReplyDelete
  153. the very foundation for science, that everything from a proton to a platypus has certain characteristics and behaves a certain way, just IS formal and final causality.

    Well, if Aristotle holds the patent on things "having characteristics" and "behaving in a certain way" then he certainly would be a rich man today. I guess you are saying that anybody today (or ever) who regards the world in this way owes him. Makes you wonder how people managed to feed themselves in earlier times.

    More to the point, Green, I've rarely seen such a long-winded display of ignorance, and you have stiff competition around here. People who don't know the first thing about science would be advised to not open their mouths on the subject. Stick to prating about formal causes and other such vaporizing, it's much safer, since there's no possibility of being proved right or wrong, and apparently some people can earn a nice living at it.

    ReplyDelete
  154. People that often accuse in order to intimidate should be advised to at least try to give some arguments or to understand the subject at hand XD.



    ReplyDelete
  155. long-winded display of ignorance

    You have mischaracterized your comment--it is short-winded.

    ReplyDelete
  156. Dear Anon,

    Define "prove."

    Love,
    Me

    ReplyDelete
  157. 1. The evidence or argument that compels the mind to accept an assertion as true.

    2.
    a. The validation of a proposition by application of specified rules, as of induction or deduction, to assumptions, axioms, and sequentially derived conclusions.
    b. A statement or argument used in such a validation.

    3.
    a. Convincing or persuasive demonstration:
    b. The state of being convinced or persuaded by consideration of evidence.

    4. Determination of the quality of something by testing;

    ReplyDelete
  158. Can't tell if Eduardo is joking or not...

    heh

    ReplyDelete
  159. No need to wait for Anon to answer anything, if everybody here is so shitty in their knowledge of Science, why he only answered when Mr Green tried to make a point that science depends on Aritotelean metaphysics, or was inspired by it somehow.

    Seems fairly odd, considering that I, Sean, Reighley ... damn most of us here talk about Science one way or another.

    Perhaps that was just the drop for him/her.

    ReplyDelete
  160. No Joe, XD I just looked for the definition of the work proof... or prove.

    And that is what I found XD. Shame is that ... apprently arguments work as proves/proofs as well 8D!

    ReplyDelete
  161. Green,

    Thanks a ton for that writeup. I've heard similar sentiments before, but that's just a damn fine way to have it presented.

    ReplyDelete
  162. Dguller

    You can read about Aristotle’s denunciation of experimentation in David Lindberg’s The Beginnings of Western Science on page 51, which you can read online on Google Books. He cites Aristotle’s Politics as support for this: “But then we must look for the intentions of nature in things which retain their nature, and not in things which are corrupted.” For Aristotle, “corruption” occurs when something is prevented from actualizing its final ends, which presumably he thought would occur outside of its natural environment.



    That sounds like a stretch.

    I do not think Aristotle meant it as an opposition to experimentation

    Actually Aristotle point is correct, i.e. things should be eximined PROPERLY, in their correct 'state', not in an artificial state that will lead you to wrong conclusions.

    Indeed this is a problem of science: when do the interpretation of the experimental results describe reality correctly?

    This is a problem especially in biology, where results in vitro and in vivo might have a jarring contrast.

    Ao, although Aristotle probably was not considering modern expetimental methods, he sure does make an important point that scientists need to consider.

    ReplyDelete
  163. lol that is a goo point Ismael.

    I remember in the very beginning of a book a researcher says that at first you must choose which is the proper way to conduct your research if either you will only observe and describe, or you will do a series of experiments.

    I never thought that such a simples difference in my opinion could open a avenue to criticism of an experiment! Although I have seem that happen before just never interpreted things that way!

    ReplyDelete
  164. Maybe I should change my name to: Lord Typo.

    xD

    ReplyDelete
  165. Rank:

    Personally, I have appreciated our discussions on this blog, as exhausting as they have been. Although I have learned a great deal from Feser and other commenters here, our marathon-like dialogues here have helped me clarify my ideas the most, and you are an excellent interlocutor. It is a shame that you will be posting less here, because your thoughts are always worth reading, but I understand that there are more important things in life. If in your future readings you come across information that sheds light upon our previous discussions regarding the paradox of prime matter, whether the intellect limits forms after receiving them, and whether a being can have more esse than another, and thus be more good than another, then mention them here, because that would really help me out. Anyway, good luck, my friend.

    ReplyDelete
  166. Oh don't be so damn jellysifh... just exchange emails XD.

    ReplyDelete
  167. @ Reighley

    The Mother of All Algorithms would be a very different kind of beast from all its daughters, including programs that analyse and prove numeric and logical theorems (presented to the program as strings of logical and numeric characters), and programs that produce other programs, according to the programmers' logical rules.

    The Mother of All Algorithms would take the User's Specification as input, and produce a usable program as output. To do this it would have to be able to process both intention and intentionality as inputs.

    ReplyDelete
  168. @seanrobsville,

    "The Mother of All Algorithms would take the User's Specification as input, and produce a usable program as output. To do this it would have to be able to process both intention and intentionality as inputs."

    Is it expected to read my mind? Or may I tell it what I want in English? Because English is just a finite string of symbols too. Numerical representation by another name.

    Anyhow I think you and I agree, though for different reasons that this Mother of All Algorithms is impossible. Let us avoid the counterfactual. Why doesn't your argument work for theorem provers?

    ReplyDelete
  169. Are the algorithms really proving theorems or telling you that it has proven a certain theorem and you read off the output that?

    ReplyDelete
  170. Theorem provers operate in the currency of computation - logical and numerical values and operations corresponding to the computer's instruction set and binary memory. At no point do they have to exchange this currency for real world purchases - desired outcomes such as a landing on Mars.

    'English is just a finite string of symbols too. Numerical representation by another name.'

    But what is represented is not the currency of computation. 'Take this probe to Mars and land it without damage' is a User Requirements Specification which has no numerical content.

    ReplyDelete
  171. Sean,

    You [December 2, 2012 7:28 AM]: A single formula can have multiple uses because no computational model of the mind (or attempted implementation of such a model) contains within itself any intentionality. All such meaning must be projected on to the data and procedures from 'outside the system'. You can have syntax but not semantics within such a model (Chinese Room). Now if we assume that computationalism is equivalent to physicalism (Church-Turing-Deutsch principle), we can similarly critique any mechanistic model or explanation of the mind.

    Me: [December 2, 2012 8:17 AM] if we assume that computationalism is equivalent to physicalism (Church-Turing-Deutsch principle), we can similarly critique any mechanistic model or explanation of the mind. I think more would be needed--if I understand it correctly, the simplified, basic claim wouldn't be that intentionality comes from algorithms which themselves are devoid of intentionality, but that it emerges from the interaction of said algorithms, i.e., it would be loosely akin to a chemical reaction between two inert substances.

    You: [December 2, 2012 9:10 AM]@Glenn "I think more would be needed--if I understand it correctly, the simplified, basic claim wouldn't be that intentionality comes from algorithms which themselves are devoid of intentionality, but that it emerges from the interaction of said algorithms, i.e., it would be loosely akin to a chemical reaction between two inert substances." Emergent phenomena arise in the mind of the observer. Emergence presupposes the pre-existence of mind and intentionality... http://seanrobsville.blogspot.com/2009/10/consciousness-and-mind-as-emergent.html

    Let's do away with the "simplified, basic claim", and hone in on something a bit more to the point (no pun intended). From COMPUTATIONALISM UNDER ATTACK:

    (cont)

    ReplyDelete
  172. As pointed out above, many arguments against computationalism are based on disagreements concerning computational architecture. The same could be said of the format of representations. Sometimes, computationalism is identified with the choice of a particular kind of representation, typically "language-like" representations characteristic of classic AI and cognitive science (logic-based representations, production rules, semantic networks, frames, and so on). These representations are processed by explicit manipulation rules. Representations with a less "linguistic" structure (firstly, distributed or "subsymbolic" connectionist representations) have been considered less akin to computationalism. These claims, too, usually stem from a restricted view of computationalism. Computationalism, per se, is not compromised by any particular kind of representation or process [my emphasis] (once accepted that they are effective processes). Frequently, these disputes come from confusion concerning different levels of explanation. The analysis of the levels of explanation in cognitive science developed by David Marr may be useful here.

    According to Marr (1982, chapter 1 [Marr, D. (1982). Vision, Freeman, New York]), a computational explanation can be stated at three different levels, the
    level of the computational theory, the algorithmic level and the implementation level. The level of the computational theory is the most abstract; it is concerned with the specification of the task of a certain cognitive phenomenon. At this level, cognitive tasks are characterised only in terms of their input, their output, and the goal of the computation, without any reference to specific cognitive processes and mechanisms. In other words, at the level of computational theory a cognitive task is accounted for in terms of a functional mapping between inputs and outputs. The algorithmic and the implementation levels deal, at different levels of abstraction, with the specification of the task identified at the computational level. The algorithmic level explains "how" a certain task is carried out: it deals with the computational processes and with the processed data structures (i.e., the "representations"). The implementation level deals with the physical features of the device (e.g., neural structures) implementing the data structure and the procedures singled out at the algorithmic level. The relationship between computational theory and algorithmic level is the same as that existing between a mathematical function and an algorithm that computes its values.

    The aim of a computational theory is the individuation of a (computable) function
    f as a model of a given cognitive phenomenon. At the computational theory level, no assumption is made on the algorithms that compute f, nor, a fortiori, on their implementation. The role of the computational level is to allow a more abstract understanding of cognitive phenomena: the computational explanation of a cognitive phenomenon cannot be reduced to the exhibition of an algorithm [my emphasis] (or, worse, a computer program) that simulates its behaviour (as happens, for Marr, in many alleged cognitive models developed in AI).

    It can be said, then, that thinking that attacking (e.g.) the absence of "aboutness" from algorithms will somehow bring down computationalism is akin to thinking a helicopter can be brought down by shooting a BB gun at the air beneath it.

    This isn't to defend computationalism, but to suggest that if you want to (successfully) critique it, something with a few more teeth than your current approach is called for.

    ReplyDelete
  173. I don’t think anyone endorsing scientism would say that only artificial experimentation is appropriate to understand the natural world. There is an important role for observation and experimentation both under the rubric of scientism.

    Under the rubric of science perhaps. Not scientism. There's a big difference.

    Plenty of times I have been involved in debates with the believers of scientism only to witness them resort to nonsensical claims in the nature of, if it's not backed up by scientific experimentation then it isn't "real".

    In fact I had an exchange with someone 3 days ago, where he did precisely that.

    ReplyDelete
  174. Must be really tough to get an experiment started with that guy ... nothing exists, therefore I get a bit of nothing and test against this other nothing and then I get some nothing out of it, and then do some nothing and voilá nothing!

    Thanks Pop Science, thank you for making people that love science into complete idiotic ideologues with nothing in their minds but pre-fab phrases! THANKS.

    * Yeah ... I happen to study Physics to be a teacher or professor someday and those things are just a bowl of puuuure happiness for me *

    ReplyDelete
  175. More to the point, Green, I've rarely seen such a long-winded display of ignorance, and you have stiff competition around here. People who don't know the first thing about science would be advised to not open their mouths on the subject.

    On the contrary, the only one here who doesn't understand science is you, if you think what Green said is in fact false. You are prime example of what I call the cancer on science. People, who neither understand its nature, limits, history and dependence on both language and metaphysics that are best advised not to speak of science as if they understand it. Your words are nothing more than mental pollution.

    If you're going to come here and talk nonsense at least try to justify your claims. If you can't (which in this case you can't since what you're advocating is a failure of epic proportions) at least open your ears and eyes, because you might actually learn something.

    ReplyDelete
  176. @Black Luster

    That's great to hear. I still have my doubts, but I think Feser's case is solid. There was a point where I was essentially a nihilist, but then I discovered WLC's work. From there I started roaming the blogosphere, and I found Pruss, Reppert, Feser, Vallicella and others. They shaped my outlook in life, but I still have to read the rest of their stuff. Probably during winter break.

    I too came to philosophy from a skeptical standpoint. I use to be a naïve materialist prior to asking some very important questions about the world and of myself in regards to who I was, and more importantly, who I wanted to be. Doubt is part of faith, that’s why it’s called faith and not certainty (if such thing really exists) and it’s an integral part I believe in remind us to be humble.

    The names you mentioned I certainly the same names that I run into in my philosophical investigations (no, I am not Ludwig Wittgenstein’s spirit :-P). I’m taking the liberty to recommend a few more for you as I have found them to be very helpful. My approach is to hear what everyone has to say and not approach the issue from one perspective only (e.g. Only Aristotelian, or only post-modern etc). Authors I would recommend reading for enhancing your understanding of not only Theological matters but philosophy and science as well as metaphysics in general would be:

    Stephen Barr: “Modern Science and Ancient Faith”
    Anthony Rizzi: “The Science Before Science”
    John Polkinghorne: Anything he’s written
    Mortimer Adler: “How to Think About God”
    Various Authors: Blackwell Companion to Natural Theology
    G.K. Chesterton: (anything he’s written, I especially enjoyed his humorous critique in Heretics)
    Owen Barfield: Saving the Appearances (it has the potential to shift the way you think about thought – highly recommended) and Worlds Apart
    CS Lewis: Anything he’s written
    Michael Polanyi: Personal Knowledge (One of the best philosophers of science ever!)
    Paul Feyerabend + Thomas Khun’s seminal, critical books on science (Against Method and Structure of Scientific Revolutions)
    Norman Geisler: Christian Apologetics (has some very unique and interesting ideas)
    Works by Greg Bahsen, James Anderson, John Frame and Van Til.
    Works by Plantinga and Swimburne on God, epistemology, ontology etc.
    Bruce Rosenblum: Quantum Enigma (It’s basically a science book that also touches on some metaphysical issues that I found to be very simple and interesting). Once you’re done with that any work by Ian Thompson who writes on quantum theory from what appears to be an Aristotelian perspective.
    I would also recommend reading at least some Descartes, Hume, Hagel, Kant and nietzsche. They will help you understand the development of modern thought and the eventual problems of nihilism that face much of the world (both intellectual and every day worlds) today.
    Saul Kripke: “Naming and Necessity”
    David Oderberg: Real Essentialism
    Finally, some Kiergegaard, Paul Tillich and of course David Bentley Hart (Book: The Beauty of the Infinite), who I am currently very fascinated by.

    There are a lot more books and articles I’ve read over time, but these are the ones that just popped in my head. Not all of them are in unison. In other words, some might propose ideas that might be in conflict with ideas from other books I mentioned but I believe it’s good to have variety and to know how different people approached the issue. If anything, you’ll experience the wealth of ideas that Theists have offered over the last 2500+ years. As someone who understood the world materialistically, I can say with confidence that reading all these thinkers blew my mind away and illuminated my reality in ways that were previously unimaginable!

    It’s as if my world went from being black and white to full color.


    If you want I can suggest more.

    ReplyDelete
  177. Know what... F all of you and your awesome knowlege of philosophy XD. U_U the only philosophy book I have ever tried to read was a Bertrand Russell's book about epistemology. I read 5 pages and brought the book back to it's shelve XD.

    5 pages were enough for me to know that I completely disagreed with him on how we gain knowledge and that he was some kind of thinker that tries to shrink epistemology as much as he can.

    Oh good times when I use to be all happy about the possibility of reading the Institute's entire Library n_n!

    ReplyDelete
  178. @Glenn

    If you were to critique computationalism, how would you go about it? What do you think is the best argument against it?

    ReplyDelete
  179. Anon,

    That list is pretty amazing. I see some familiar names, though I haven't read them yet. There's some stuff I want to check out asap, in particular Oderberg, Hart and CS Lewis, as well as Feser's Aquinas. Now I just need to somehow get cash for those. I guess I'll have to wait a bit and sell some textbooks once the semester is over.

    ReplyDelete
  180. Eduardo,

    Wisdom and courage are two virtues. To gain philosophical wisdom you must first have the courage to open up and read boring and confusing philosophy books.

    ReplyDelete
  181. I was asked to expand on my earlier mocking of Green's scientific ignorance. Here you go:

    If Aristotle is so bad for science, why was it when Aristotelian-based philosophy reached its highest point that the seed of modern science was planted?

    Or, you could ask the far less ridiculous question, why didn't science originate during the thousand-year reign of Aristotle, but only came into being when Bacon and others found the courage to reject him?

    Anyway, all that railing against the establishment didn't stop Modern Science from spending daddy's money. It always talked about rejecting formal and final causes, but as article after articles on this site makes clear, you can't have basic physics without both.

    You would be well-advised not to draw any lessons about basic physics, or anything else, from articles on this site.

    Of course, the "teleology" that was rejected was in large part stuff like "polar bears have white fur so they can hide in the snow", which doesn't help you breed genetically-altered bears to sell designer furs.

    Oh for god's sake, how confused can you be? What makes you think that modern biology has trouble acknowledging that the white fur of the polar bear is an adaptation to its environment? This is evolution 101 and if you don't understand it, then you understand nothing.

    So that level of biological teleology was rejected in favour of chemical and physical teleology which obviously doesn't dispense with finality in any way, but merely shows more interest in one kind than in another.

    WTF is "chemical teleology"? Biology has teleology (not as any magical sauce ladeled over reality, but simply as an emergent property of differential reproduction with mutation), chemistry and physics, as far as we know, do not.


    So it's not just that Aristotelian-type philosophy "vs." science is a false dichotomy; it isn't even really Aristotelian-type science vs. modern science — the very foundation for science, that everything from a proton to a platypus has certain characteristics and behaves a certain way, just IS formal and final causality.

    As I said earlier, the notion that Aristotle has some kind of patent on things having characteristics and behaviors is one of the stupidest things I've ever heard.

    Do you think that people before Aristotle were incapable of thinking and talking about things, classes of things, characteristics of things, and behaviors of things? The Sumerians and Babylonians managed to build some serious early civilizations, a little hard to imagine how they could do that if they were not capable of thinking in this elementary way.

    And physics itself has had problems, like the indomitable billiard-ballism that plagues QM.

    Seriously, WTF? Physics certainly has problems, but "billiard-ballism" is not one of them. Now, some undergraduates trying to learn QM might suffer from "billiard-ballism", if you define it as trying to apply a billiard-ball model inappropriately, but this is not a problem with physics as practiced by professionals.

    But you'll end up finding it all much "spookier" than Aristotle would have.

    If the implication is that Aristotle would not have any problem comprehending quantum mechanics, you are daft.


    And of course, we got all these wacky problems concerning the mind: like the "hard problem" of how to get intentionality out of non-intentioal parts, instead of the easy problem of how to get intentionality out of intentional parts.

    So the mind is composed out of "intentional parts"? What are they?

    Of course, this is all itself a very caricatured and over-simplified view of a vastly complex series of historical events...

    That's putting it mildly.

    ReplyDelete
  182. Anonymous,

    @Glenn
    If you were to critique computationalism, how would you go about it? What do you think is the best argument against it?


    I haven't any desire to challenge or critique computationalism. I also don't know what might be the best argument against it.

    But here is an excerpt from Gualtiero Piccinini's Computationalism in the Philosophy of Mind, somewhat in keeping with what was emphasized in the quotation included in my earlier comment:

    Computationalism is also sometimes confused with the stronger view that mental states are computational states, in the sense that their nature is wholly (as opposed to partially or not at all) computational. This stronger view, which may be called 'computational functionalism', is not very popular, mostly because of worries about accommodating consciousness. Although there are philosophers who have given computational accounts of consciousness (Lycan; Dennett; Rey), many others doubt that simply being a computational state is enough for being a conscious state (e.g., Block). But rejecting the view that mental states are purely computational is consistent with the acceptance of computationalism [my emphasis]: even if consciousness involves more than computation, computation may still explain behavior (in whole or in part).

    Again, I'm not defending computationalism (see Section 8 of CPM for some objections against computationalism); just showing that the legs Sean thinks he is cutting out from under (the non-bastardized version of) computationalism aren't the legs that it's standing on.

    ReplyDelete
  183. @anon

    WTF is "chemical teleology"? Biology has teleology (not as any magical sauce ladeled over reality, but simply as an emergent property of differential reproduction with mutation), chemistry and physics, as far as we know, do not.

    I told you that you're ignorant and instead of listening you went ahead and proved me right. This thing you call an emergent property is just another word for formal cause. Now thank Aristotle and be quiet before you embarrass yourself even more.

    Also, if you don't think that quantum theory has literally destroyed the materialist conception of elementary particles, you're only deluding yourself. Even Heisenberg likened QM to Aristotelian concepts such as potentia.

    I will not bother with the nonsense included in the rest of your post. Maybe someone else has the patience for it.

    ReplyDelete
  184. "I was asked to expand on my earlier mocking of Green's scientific ignorance. Here you go:"

    Try harder. Science is fine, Mr. Green is adressing its metaphysical basis and the effects of that metaphysical basis on our understanding.

    "So the mind is composed out of "intentional parts"? What are they?"

    This cannot be explained in the combox. Start with this:

    http://edwardfeser.blogspot.com/2011/05/mind-body-problem-roundup.html

    If you have not done so, read through the comments in this post from the beginning.

    ReplyDelete
  185. "If you have not done so, read through the comments in this post from the beginning."

    And by this I am referring to this very post, not the ones under the mind-body roundup.

    ReplyDelete
  186. Glenn: You may also make fun of, sneer at, mock, disparage and otherwise show and express contempt for the Aristotelian notion of forms, as well as Aristotelian logic

    I think that the Anonymous who posted after you took your "you may" a bit too literally, given his performance. (Per-FORM-ance— ...ah, never mind.)

    Crude and others: thank you! (I only wish some anonymous troll had bothered to in-FORM me that I didn't know anything about science before I spent all that time writing it... in fact, it turns out that I was wrong, and formyl causes were actually discovered by Antistotle. So there.)

    ReplyDelete
  187. Well I suppose we are back at the point where the critic has no idea what he is talking about.

    He is stating his metaphysical conclusions based on A-T philosophy, the teleology he speaks of is not really the one you have in mind.

    The implication of what he is saying is that someone going about QM pressuposing Aristotle metaphysics wouldn't find any of that "spooky".

    Mr Green is talking about the metaphysical pre-assumptions scientists make, this may not pose any problem while analysing the data though. It is sort of a tangent subject.

    Apparently Mr Green does make some rather hardcore assertions and if he feels compelled to clear them is up to him, but Anon you don't seem to know what people are talking about here.

    ReplyDelete
  188. Glenn

    aparently computationalism, what I think they are talking about, is the idea that the mind is the brain or part of it and it is no different from a computer.

    Is juse WAYYY more complex wiring but no different from a computer.

    ReplyDelete
  189. Anonymous: I was asked to expand on my earlier mocking of Green's scientific ignorance. Here you go:

    Nono, you misunderstand. (Now there's a shocker!) You were supposed to engage in further mockery, which at least has a modicum of entertainment value, as opposed to trying to explain stuff, which is just depressing.

    WTF is "chemical teleology"?

    If you understood anything about Aristotelian metaphysics, you'd know; but of course, you don't. Which leads one to wonder why you would claim there isn't any. "I don't know what you're saying or why, I just know that you're wrong!" Look, if you want to take the word and make up a new meaning for it, well, knock yourself out; but you can't completely ignore what Aristotelians mean by it and then say that they must be wrong.

    As I said earlier, the notion that Aristotle has some kind of patent on things having characteristics and behaviors is one of the stupidest things I've ever heard.

    It is pretty stupid. And yet you're the only one saying it (twice, now). But of course the whole point is that Aristotle didn't invent this stuff; he merely observed the world around him. Anyone else could do it and reach the same conclusions. That's why science succeeded: because it accepted the same facts about reality that Aristotle did. And yet you are claiming that (a) Aristotle's ideas are such blatant facts that we shouldn't even associate his name with them, and (b) that by rejecting these facts, Bacon & co. invented Science. Um. Yeah.

    ReplyDelete
  190. "in fact, it turns out that I was wrong, and formyl causes were actually discovered by Antistotle. So there"

    Oh God I died. Im such a nerd. Mr. Green is on a roll.

    ReplyDelete
  191. Well guys we have like a week to the end of the world, and I gotta a test to make XD, very dangerous very important...

    ReplyDelete
  192. Eduardo,

    The computationalism you're thinking of, which holds that the mind is no different from a computer, is a vulgar knock-off of what computationalism started out as. That it has been metastasized this way and that, and that it has been co-opted and pressed into service for something it never was intended for is a knock against the metastasizers, not computationalism itself. At its highest, most abstract level, it is supposed to be about coming up with descriptions of what is commonly taken to be intelligent behavior, and not about explaining the mind or accounting for consciousness.

    Re 'the mind is what the brain does', I personally have not a problem with this being employed as a basic rule of thumb meant to indicate that the workings of our minds are influenced by our brains.

    What I do have a problem with is when, as is so very commonly the case, 'the mind is what the brain does' is employed as a shortened version of the "mind is only, solely, and nothing but what the brain does", i.e., when mind and brain are taken to be synonymous.

    Good luck with your test.

    ReplyDelete
  193. Anonymous,

    Or, you could ask the far less ridiculous question, why didn't science originate during the thousand-year reign of Aristotle, but only came into being when Bacon and others found the courage to reject him?

    Although this is--subsequent to its having been cleansed of its emotive content--a good question, I wonder whether you may be interested in a genuine response, or are simply content with having hastily scored what you may possibly take to be a valid implied and disparaging point. I will pretend to be an optimist, and assume, against my better judgment, that you may be interested in a genuine response.

    First, I'll roll up my sleeves--all the way beyond the elbows, and up to the forearms like this--so that you may easily see that I've nothing hidden there. Next, I'll take three steps back. And, last, I will let Bacon himself take the stage.

    As to why "what we [Bacon and Co.] now offer should have so long been concealed from, and have escaped the notice of mankind", and that "it should even now have entered any one's mind, or become the subject of his thoughts", the reason is, says Bacon, "the gift of fortune [rather] than of any extraordinary talent" and "the offspring of time rather than wit".

    Bacon then gets a little more specific in listing four causes:

    o "The first cause, then, of such insignificant [prior] progress in the sciences, is rightly referred to the small proportion of time which has been favorable thereto." (The "small proportion of time which has been favorable thereto" is guesstimated by Bacon to have been about six centuries. (I'm not aware of him having later performed any experiments in an attempt to confirm or refute his guesstimate.))

    o "A second cause offers itself, which is certainly of the greatest importance; namely, that in those very ages in which men's wit and literature flourished considerably, or even moderately, but a small part of their industry was bestowed on natural philosophy, the great mother of the sciences."

    o "Add to this that natural philosophy, especially of late, has seldom gained exclusive possession of an individual free from all other pursuits, even among those who have applied themselves to it[.]"

    o "There is another powerful and great cause of the little advancement of the sciences, which is this; it is impossible to advance properly in the course when the goal is not properly fixed. But the real and legitimate goal of the sciences is the endowment of human life with new inventions and riches."

    That last cause is especially telling.

    ReplyDelete
  194. Mr. Green,

    I think that the Anonymous who posted after you took your "you may" a bit too literally...

    After posting it, my heart skipped a beat: "Oh no, the last clause can easily be construed as an invitation..." My apologies to you, and readers, for the negligence.

    ReplyDelete
  195. Damn oscilations and waves XD. It looks simple in real life until you try formulize it with careful mathematical formulas XD.

    Didn't know computationalism started out as something else... well not a surprise there. But that is rather interesting, much like my professor's dialectical materialism... yeah physics is filled with crazy people, that is why I love it.

    Well personally I also never had a problem with mixing them together at first I use to try to theorize people's intentions to explain people's answer to certain things and I felt that it sort of made sense to give the brain an rule-based operation and the mind a more subjective operation but still based in some causal rules.

    Or course being the lazy bum as I am, I never kept trying it out to see where it would lead me...

    ReplyDelete
  196. Here's a silly question which is also thought provoking:

    If the brain itself were sufficient to explain the mind, how is it that 5-yr olds can outthink cadavers?

    What the heck, it's late; silliness rules, so here's a joke:

    What's the difference between phrenology and neuroscience?

    Phrenology concerns itself with nodules on the head, while neuroscience concerns itself with modules in the head.

    ReplyDelete
  197. Well because the mind is ......... errr.... in the bosons of our neurons... it is .... err.... the field... no wait ... it .... is .... FUCK YOU!

    t_t

    Now one thing I found very funny is how I while trying to explain or think of an explanation for mind or consciousness took 3 days to think, well if there is a explanation in thw way I am thinking then it must be dues to the relation among the parts and not because of the parts itself.

    And some people take that conclusion as A HUGE breakthrough!!!!... seriously, I had that breakthrough while playing shooters man.

    * not trying to point thingers at any monist around here, but rather at a certain experience I had in the past XD and it was internally funny to me... *

    ReplyDelete
  198. Did I just wrote fingers with TH... Oh shoot!

    ReplyDelete
  199. * not trying to point thingers at any monist around here, but rather at a certain experience I had in the past XD and it was internally funny to me... *

    I can well imagine how it might be. Food for thought: were there multiple observers of the various parts interacting--or just one observer?

    ReplyDelete